Sawaal

  • Aptitude and Reasoning
  • General Knowledge
  • Certifications
  • True or False
  • Submit Q & A
  • Aptitude and Reasoning /
  • Non Verbal Reasoning /
  • Analytical Reasoning

 Analytical Reasoning Questions

Find the number of triangles in the given figure.

analytical reasoning problem solving

The simplest triangles are AML, LRK, KWD, DWJ, JXI, IYC, CYH, HTG, GOB, BOF, FNE and EMA i.e. 12 in number.

The triangles composed of two components each are AEL, KDJ, HIC and FBG i.e. 4 in number.

The triangles composed of three components each are APF, EQB, BQH, GVC, CVJ, IUD, DUL and KPA i.e. 8 in number.

The triangles composed of six components each are ASB, BSG, CSD, DSA, AKF, EBH, GGJ and IDL i.e. 8 in number.

The triangles composed of twelve components each are ADB, ABC, BCD and CDA i.e. 4 in number.

Total number of triangles in the figure = 12 + 4 + 8 + 8 + 4 = 36.

View Answer Report Error Discuss Filed Under: Analytical Reasoning

Find the minimum number of straight lines required to make the given figure.

analytical reasoning problem solving

The horizontal lines are AK, BJ, CI, DH and EG i.e. 5 in number.

The vertical lines are AE, LF and KG i.e. 3 in number.

The slanting lines are LC, CF, FI, LI, EK and AG i.e. 6 in number.

Thus, there are 5 + 3 + 6 = 14 straight lines in the figure.

Choose the alternative which is closely resembles the water image of the given combination/figure.

download_(1)1492407904.png image

View Answer Report Error Discuss Filed Under: Analytical Reasoning Exam Prep: AIEEE , Bank Exams , CAT , GRE , TOEFL Job Role: Bank Clerk , Bank PO

What is the number of straight lines and the number of triangles in the given figure.

analytical reasoning problem solving

The Horizontal lines are DF and BC i.e. 2 in number.

The Vertical lines are DG, AH and FI i.e. 3 in number.

The Slanting lines are AB, AC, BF and DC i.e. 4 in number.

Thus, there are 2 + 3 + 4 = 9 straight lines in the figure.

Now, we shall count the number of triangles in the figure.

The simplest triangles are ADE, AEF, DEK, EFK, DJK, FLK, DJB, FLC, BJG and LIC i.e. 10 in number.

The triangles composed of two components each are ADF, AFK, DFK, ADK, DKB, FCK, BKH, KHC, DGB and FIC i.e. 10 in number.

The triangles composed of three components each are DFJ and DFL i.e. 2 in number.

The triangles composed of four components each are ABK, ACK, BFI, CDG, DFB, DFC and BKC i.e. 7 in number.

The triangles composed of six components each are ABH, ACH, ABF, ACD, BFC and CDB i.e. 6 in number.

There is only one triangle i.e. ABC composed of twelve components.

There are 10 + 10 + 2 + 7 + 6+ 1 = 36 triangles in the figure.

What is the number of triangles that can be formed whose vertices are the vertices of an octagon but have only one side common with that of octagon?

analytical reasoning problem solving

When the triangles are drawn in an octagon with vertices same as those of the octagon and having one side common to that of the octagon, the figure will appear as shown in (Fig. 1).

analytical reasoning problem solving

Now, we shall first consider the triangles having only one side AB common with octagon ABCDEFGH and having vertices common with the octagon (See Fig. 2).Such triangles are ABD, ABE, ABF and ABG i.e. 4 in number.

analytical reasoning problem solving

Similarly, the triangles having only one side BC common with the octagon and also having vertices common with the octagon are BCE, BCF, BCG and BCH (as shown in Fig. 3). i.e. There are 4 such triangles.

This way, we have 4 triangles for each side of the octagon. Thus, there are 8 x 4 = 32 such triangles.

Count the number of triangles and squares in the given figure.

analytical reasoning problem solving

The figure may be labelled as shown 

analytical reasoning problem solving

Triangles :

The Simplest triangles are BGM, GHM, HAM, ABM, GIN, IJN, JHN, HGN, IKO, KLO, LJO, JIO, KDP, DEP, ELP, LKP, BCD and AFE i.e 18 in number

The triangles composed of two components each are ABG, BGH, GHA, HAB, HGI, GIJ, IJH, JHG, JIK, IKL, KLJ,LJI, LKD, KDE, DEL and ELK i.e 16 in number.

The triangles composed of four components each are BHI, GJK, ILD, AGJ, HIL and JKE i.e 6 in number.

Total number of triangles in the figure = 18 + 16 + 6 =40.

The Squares composed of two components each are MGNH, NIOJ, and OKPL i.e 3 in number

The Squares composed of four components each are BGHA, GIJH, IKJL and KDEL i.e 4 in number

Total number of squares in the figure = 3 + 4 =7

View Answer Report Error Discuss Filed Under: Analytical Reasoning Exam Prep: AIEEE , Bank Exams , CAT Job Role: Bank Clerk , Bank PO

Find the number of triangles in the given figure ?

analyticalreasoning81484544890.png image

The simplest triangles are AKI, AIL, EKD, LFB, DJC, DKJ, KIJ, ILJ, JLB, BJC, DHC and BCG i.e. 12 in number. The triangles composed of two components each are AKJ, ALJ, AKL, ADJ, AJB and DBC i.e. 6 in number. The triangles composed of the three components each are ADC and ABC i.e. 2 in number. There is only one triangle i.e. ADB composed of four components. Thus, there are 12 + 6 + 2 + 1 = 21 triangles in the figure.

View Answer Report Error Discuss Filed Under: Analytical Reasoning Exam Prep: Bank Exams , CAT Job Role: Bank Clerk , Bank PO

analyticalreasoning1331484546669.png image

The given figure can be labelled as :

analyticalreasoning13answer121484546728.png image

The simplest triangles are AJF, FBG, HDI, GCH and JEI i.e 5 in number. The triangles composed of the three components each are AIC, FCE, ADG, EBH and DJB i.e 5 in number. Thus, there are 5 + 5 = 10 triangles in the given figure.

Quick Links

Advertise with Us

Terms of Use

Privacy Policy

Study Material

Copyright © 2024 Sawaal.com | All Rights Reserved

Sawaal

If you're seeing this message, it means we're having trouble loading external resources on our website.

If you're behind a web filter, please make sure that the domains *.kastatic.org and *.kasandbox.org are unblocked.

To log in and use all the features of Khan Academy, please enable JavaScript in your browser.

Course: LSAT   >   Unit 1

  • Getting started with Analytical Reasoning
  • How to approach ordering setups
  • How to approach grouping setups
  • How to approach mixed setups
  • Given info: basic orientation | Quick guide
  • Given info: could be true/false | Quick guide
  • Given info: must/cannot be true/false | Quick guide
  • New info: could be true/false | Quick guide
  • New info: must/cannot be true/false | Quick guide
  • Equivalent rule, min-max and completely determines | Quick guide
  • Equivalent rule | Learn more

Study plan for analytical reasoning | Getting more than 10 right

  • How to use multiple scenarios in analytical reasoning setups
  • Deductions in analytical reasoning | Introduction
  • Deductions in analytical reasoning | Practice
  • Diagram notation conventions for analytical reasoning setups

analytical reasoning problem solving

I’m currently getting fewer than 10 questions correct in Analytical Reasoning—help!

Your lsat growth mindset, top strategies to take your score to the next level.

  • Takes 30 minutes to work through just one setup and gets the majority of questions wrong
  • Reviews the questions she missed
  • Reads every hint and explanation
  • Reads relevant help articles
  • Watches worked example videos
  • Tries to implement new strategies on the next setup
  • Takes 10 minutes to work through a setup and gets the majority of questions wrong
  • Does another setup and gets the majority of questions wrong
  • Doesn't take the time to review because he just wants to do exercises until his score goes up.

I have no idea what I’m doing or looking at

I don’t understand these “if-then” statements that i sometimes see in the rules, i feel like i know what i’m doing, but then i get a lot of the questions wrong that i thought i got right, i think i know what i’m doing, but i can’t even make it past the second of the 4 setups in 35 minutes, skipping questions.

  • Best case scenario: You get it right! But it took you five minutes, and you didn’t even have a chance to attempt several other questions—questions that were easier. Overall, you've lost points.
  • Worst case scenario: You get it wrong, and the five minutes you spent were not only a waste of time, but you also lost possible points that you could have earned if you had been able to attempt the easier questions later in the section before running out of time.
  • Which one of the following, if substituted for the constraint that if Jaramillo is assigned to one of the ambassadorships, then so is Kanye, would have the same effect in determining the assignment of the ambassadors?
  • The assignment of parking spaces to each of the new employees is fully and uniquely determined if which one of the following is true?
  • You have been staring at the question for more than 20 seconds and still have no idea where to start.
  • It looks like you’ll need to test every choice because the answer isn’t clear from your initial diagram. For example, the question asks, “Which of the following must be true?” without giving new information, but your initial diagram doesn’t show anything that must be true.
  • You don’t understand what the question is asking you to do.
  • Mixed setups that ask you to group and sequence elements at the same time (or group 2+ sets of elements at the same time)
  • Unusual setups that aren’t straightforward ordering or grouping scenarios

Accuracy, then speed

Want to join the conversation.

Back Home

  • Search Search Search …
  • Search Search …

Analytical Thinking vs Problem Solving: A Comprehensive Comparison

Analytical Thinking vs Problem Solving

Analytical thinking and problem solving are crucial skills in various aspects of life, including personal and professional situations. While they may seem interchangeable, there are distinct differences between the two. Analytical thinking focuses on breaking down complex information into smaller, manageable components to understand a situation and evaluate alternatives effectively. On the other hand, problem solving involves devising practical solutions to overcome challenges or resolve issues that arise in daily life or the workplace.

analytical reasoning problem solving

Both analytical thinking and problem-solving skills contribute to making well-informed decisions, managing risks, and achieving success in various areas of life. By understanding these skills’ distinctions and applying them effectively, individuals can enhance their performance in the workplace, handle complex situations with ease, and make better choices in their personal lives.

Key Takeaways

  • Analytical thinking is about understanding complex situations, while problem-solving focuses on finding practical solutions.
  • Mastery of both skills leads to informed decision-making and improved risk management.
  • These abilities are essential for workplace success and overall personal growth.

Understanding Analytical Thinking

analytical reasoning problem solving

Nature of Analytical Thinking

Analytical thinking refers to a mental process in which a person systematically breaks down complex problems or situations into smaller, manageable components. This enables the identification of essential elements and their relationships, leading to an effective solution. Analytical thinkers excel in identifying patterns, interpreting data, and drawing conclusions based on factual information. Unlike reactive problem-solving, which focuses on finding immediate remedies, analytical thinking is strategic in nature, seeking long-term solutions by addressing the root causes of a problem.

Key components of analytical thinking include reasoning, fact-checking, and questioning assumptions. This skill set allows individuals to approach problems with an open mind, meticulously gather and analyze data, and make well-informed decisions. Ultimately, analytical thinking leads to more informed and strategic decision-making, increasing the likelihood of success in professional and personal endeavors.

How Analytical Thinking Works

The process of analytical thinking unfolds in several stages:

  • Identify the problem or situation : Determine the issue that needs addressing and clearly define its scope.
  • Gather relevant data : Collect information related to the problem from various sources, ensuring its accuracy and reliability.
  • Break down the problem : Dissect the problem into smaller, manageable parts to gain a better understanding of its intricacies.
  • Analyze and interpret data : Examine the data to identify patterns, trends, and relationships, and derive insights using logical reasoning.
  • Question assumptions : Challenge any preconceived notions or biases that may skew the analysis and arrive at the most objective conclusions possible.
  • Generate solutions : Propose potential solutions based on the analysis, weighing their pros and cons.

It is important to note that analytical thinking is not solely reserved for mathematicians or scientists but is a valuable skill applicable to a wide range of disciplines and professions. From business analysts, who require analytical thinking and problem-solving skills to identify and implement changes, to daily decision-making in personal lives, analytical reasoning plays a vital role in successfully navigating through various complexities.

Significance of Problem Solving

Features of problem solving.

Problem solving is an essential skill that helps individuals and organizations tackle challenges effectively. Problem-solving skills enable individuals to identify problems or obstacles, analyze the situation, and find appropriate solutions. These skills include critical thinking, analytical reasoning, decision-making, and learning from the process. People with strong problem-solving abilities can better cope with stress, handle risk, and adapt to change in a fast-paced environment.

In the context of decision-making, problem solving requires individuals to evaluate multiple options and select the one with the highest probability of success. A well-developed thinking process is crucial to identifying and analyzing creative solutions, as it helps individuals see beyond the apparent issues and delve deeper into the underlying causes.

Process of Problem Solving

The process of problem-solving typically involves several stages:

  • Identify the problem : Recognizing the issue at hand and understanding its impact on the situation.
  • Gather information : Collecting relevant data and facts that will help in understanding the problem.
  • Analyze the problem : Examining the situation, breaking it into smaller parts, and identifying the root causes.
  • Generate solutions : Brainstorming various possible solutions and evaluating their feasibility.
  • Choose the best solution : Using decision-making skills to select the most suitable solution based on available information.
  • Implement the solution : Putting the chosen solution into action and monitoring its effectiveness.
  • Evaluate and learn : Reflecting on the outcomes and learning from the experience for future problem-solving situations.

By sharpening problem-solving skills and employing an organized thinking process, individuals can enhance their abilities to overcome challenges and make informed decisions, leading to personal and professional growth.

Comparing Analytical Thinking and Problem Solving

Similarities.

Both analytical thinking and problem solving involve the process of breaking down complex situations into smaller, manageable components. In both approaches, individuals need to evaluate the information at hand, identify patterns, and derive conclusions based on the evidence. This often involves receiving feedback, adapting to new information, and adjusting one’s approach.

Moreover, practicing both analytical thinking and problem-solving techniques can lead to improved decision-making abilities. This development, in turn, translates into greater efficiency and effectiveness in personal and professional contexts.

Differences

While analytical thinking and problem solving share some similarities, they also have notable differences. Analytical thinking typically follows a linear and sequential process, whereas problem solving might involve iterative processes and creative solutions.

Analytical thinking often focuses on dissecting a situation or a problem, looking for underlying patterns, and finding ways to logically deduce solutions. On the other hand, problem solving might require a combination of analytical and creative thinking, especially when faced with novel or ambiguous challenges. Problem solvers often need to develop unique strategies and evaluate alternative solutions before settling on the most effective approach.

In conclusion, analytical thinking and problem-solving, while both essential skills, have distinct applications and methods, and their effective use can be instrumental in achieving success in various aspects of life.

Ways to Improve Both Techniques

Developing analytical thinking.

Developing analytical thinking is vital for individuals seeking to improve their problem-solving abilities. One effective strategy is to practice creative activities, such as brainstorming or solving puzzles, to challenge the brain and foster development. Engaging in these tasks allows for the creation of new connections and enhances cognitive flexibility.

Another useful approach is to focus on communication and the art of listening. Active listening enables a better understanding of various perspectives and leads to well-informed decisions. Moreover, discussing complex topics can strengthen one’s ability to analyze and evaluate information effectively.

Collaborating with others can also help individuals enhance their analytical thinking skills. By working together, people can build on each other’s strengths and overcome challenges. Additionally, they can exchange ideas and learn from different viewpoints, which may lead to innovative solutions.

Enhancing Problem Solving Skills

To enhance problem-solving skills, one must be willing to take action and embrace challenges. Tackling problems head-on allows for growth and the development of practical strategies. Regular practice is essential for refining these skills and building confidence in decision-making.

Integrating soft skills, such as empathy and adaptability, play an essential role in problem-solving. Employing these abilities can improve interpersonal communication and contribute to the formation of more effective solutions.

Utilizing a methodical approach to problem-solving can also yield positive results. Techniques like breaking down complex issues into manageable steps or generating multiple possible solutions can enable a more comprehensive analysis, increasing the likelihood of success in overcoming challenges.

Finally, don’t shy away from seeking feedback from peers and mentors. Constructive criticism can highlight areas for improvement and further facilitate the development of both analytical thinking and problem-solving skills. Remember, the key to growth lies in continuous learning and adapting to new situations with confidence and clarity.

Importance in Workplace and Career Success

Relevance in the workplace.

Analytical thinking and problem solving play crucial roles in the workplace. These skills enable employees to efficiently tackle a variety of tasks and challenges. Analytical thinking refers to gathering, organizing, and evaluating information to detect patterns and identify problems. Effective problem solving involves devising creative solutions based on these findings 1 . In the modern workplace, individuals with strong analytical thinking skills can identify issues and make well thought-out decisions that contribute to overall company success 2 .

Effective communication is an important aspect of analytical thinking and problem solving. In a professional setting, employees must often convey their findings and ideas to stakeholders, ensuring that solutions are implemented appropriately and any concerns are addressed. This communication can lead to improved collaboration, clearer goals, and faster resolution of issues 3 .

Implication for Career Success

In addition to benefitting the workplace as a whole, strong analytical thinking and problem-solving skills are critical for individual career success. These skills can help professionals stand out among their peers and demonstrate their value to their organization. Professionals who can apply analytical thinking and problem-solving techniques are viewed as being able to think critically, make decisions, and take initiative, which are all highly valued by employers 4 .

Individuals who possess these skills are often able to make more informed judgments and sound decisions. This can lead to career advancement and job stability, as they are viewed as capable and dependable. Developing analytical thinking and problem-solving abilities can also open doors to new opportunities and industries, making individuals more versatile and efficient in their careers 5 .

Role in Decision Making and Risk Management

Influence on decision making.

Analytical thinking plays a crucial role in decision making, as it involves breaking things down into their component parts and using deductive reasoning to draw conclusions from given evidence and assumptions source . This allows individuals and organizations to carefully consider the pros and cons of each option, determine the feasibility of implementing potential solutions, and weigh the costs and benefits associated with each decision.

Problem-solving, on the other hand, is an analytical method that focuses on identifying potential solutions to specific situations source , sometimes requiring personal decision-making that may involve judgments or decisions on the way to find the best outcome. Both analytical thinking and problem-solving contribute to effective decision-making processes, as they provide tools and techniques for examining different courses of action and limiting uncertainties.

Contribution to Risk Management

Risk management is a critical aspect of decision-making, as it helps organizations and individuals identify, assess, and mitigate potential risks associated with various decisions. Analytical thinking contributes to risk management by enabling decision-makers to collect and analyze data, evaluate risks and their potential consequences, and make informed decisions based on the results source .

Similarly, problem-solving assists in risk management by addressing potential challenges that may arise during the implementation of solutions, such as examining potential obstacles, resource constraints, and other factors that may impact the success of an initiative source . By combining the strengths of both analytical thinking and problem-solving, decision-makers can enhance their risk management strategies and ensure a higher probability of success in their respective decisions.

In summary, analytical thinking and problem-solving are essential tools in decision-making and risk management, as they provide the necessary framework for evaluating options, weighing potential outcomes, and identifying potential challenges. By utilizing these methods, decision-makers can make more informed choices and mitigate potential risks associated with their decisions.

Utilization in Business Analysis

Application in business analysis.

Analytical thinking and problem solving are essential skills for business analysts in their day-to-day work. They are responsible for identifying, researching, and understanding complex business problems, as well as finding effective solutions to address them. By using their analytical thinking skills, business analysts can gather, assess, and interpret data from various sources to develop a comprehensive understanding of the situation at hand [1] .

When approaching a problem, business analysts consider several key factors, such as people, processes, and technology. They employ systems thinking to understand the enterprise holistically and how all these elements interact. This mindset helps them to not only identify the root cause of a problem, but also to develop solutions that address the underlying issues effectively [2] .

Understanding Financial Data

One key area where business analysts apply their analytical and problem-solving skills is in the realm of financial data. Here, they are tasked with interpreting complex financial information to derive valuable insights and make informed decisions for the organization.

In this context, their analytical thinking skills enable business analysts to:

  • Gather relevant financial data from multiple sources
  • Identify patterns, trends, and potential issues
  • Assess the quality and accuracy of the data
  • Develop conclusions and recommendations based on the analyzed data

By employing problem-solving skills, business analysts can:

  • Understand the impact of financial data on business processes and performance
  • Identify potential areas for improvement or optimization
  • Propose and evaluate relevant solutions for financial issues [3]

Overall, business analysis relies heavily on the combination of analytical thinking and problem-solving skills to address various challenges faced by organizations. The ability to understand and interpret financial data significantly contributes to the success and growth of any enterprise.

Real Life Examples

Analytical thinking and problem solving are essential skills in both personal and professional life. They allow individuals to tackle complex issues, identify the root causes, and develop effective solutions. Let’s examine some real-life examples that emphasize the differences between these two thought processes.

In the workplace, an employee might face a challenge in increasing sales. Applying analytical thinking , the individual would gather data, identify patterns, and evaluate market trends to understand the factors impacting sales performance. With this information, they can determine which areas need improvement and develop targeted strategies to address the issue. For example, they may discover that customers are dissatisfied with the available products in a particular category, prompting changes in the company’s product offering.

On the other hand, problem-solving involves addressing specific situations, such as dealing with a dissatisfied customer. In this instance, the employee would need to rely on their experience and emotional intelligence to find a solution. They would listen to the customer’s concerns, empathize with their feelings, and proactively offer options to resolve the problem. This process may include correcting mistakes made during a transaction or offering compensation for a negative experience.

Another example can be found in the realm of personal finance. Analytical thinking would be employed to evaluate one’s financial situation and understand patterns in spending habits. This analysis could reveal areas where money may be saved or better utilized. For instance, it may uncover excessive spending on dining out or ineffective monthly budgeting practices.

Conversely, problem-solving can come into play when an unexpected financial emergency occurs. In such cases, one would need to quickly evaluate the situation and devise creative solutions to address the crisis. This might involve temporarily reducing non-essential expenses, seeking additional sources of income, or negotiating payment plans with creditors.

In both of these real-life scenarios, analytical thinking and problem-solving work in tandem, complementing each other to achieve effective outcomes. While individuals may favor one approach over the other, it is crucial to recognize and develop both skillsets to navigate the complexities of modern life successfully.

  • https://www.glassdoor.com/blog/guide/analytical-thinking/ ↩
  • https://www.indeed.com/career-advice/career-development/problem-solving-and-decision-making ↩
  • https://www.radford.edu/content/cobe/innovation-analytics/analytics/career-prep/report-e.html ↩
  • https://www.indeed.com/career-advice/career-development/critical-thinking-vs-problem-solving ↩
  • https://www.amanet.org/analytical-thinking-problem-solving-and-decision-making/ ↩

You may also like

analytical reasoning problem solving

Are Problem Solving and Critical Thinking the Same? Debunking the Common Misconception

Problem solving and critical thinking are often considered synonymous, but they are two separate skills with distinct strategies, purposes, and applications. Understanding […]

When To Use Critical Thinking

When To Use Critical Thinking – What You Need To Know

There is a time and a place to use different skills in life. When faced with a problem that requires calm, logical […]

communication and critical thinking

Critical Thinking and Effective Communication: Enhancing Interpersonal Skills for Success

In today’s fast-paced world, effective communication and critical thinking have become increasingly important skills for both personal and professional success. Critical thinking […]

Critical Thinking and Time-Management

Critical Thinking and Time-Management (How to get the most important things done)

Critical thinking and time management go together. Time management is really just a problem that needs to be solved, and that problem […]

Free Analytical Reasoning Test Example Questions and Answers [2024]

Each practice test contains detailed answers and explanations

  • 32 Numerical Reasoning Practice Tests.
  • 15 Verbal Reasoning Practice Tests.
  • 30 Figural Reasoning Practice Tests.
  • Aptitude Tests
  • Analytical Reasoning Tests

Are you required to take an Analytical Reasoning test as part of your hiring process? JobTestPrep offers sophisticated practice materials that can increase your chances of excelling in any Analytical Reasoning pre-employment assessment.

The Analytical Reasoning Test (Also known as the analytical skills test) practice preparation includes the following:

  • Numerical Reasoning - This section includes 32 practice tests on topics such as Algebra, Math Word problems, Number Series, Numerical Reasoning, Table, and Graphs that will increase your math knowledge and solution speed.
  • Verbal Reasoning - This part includes 9 true\false\cannot say, 4 logical reasoning, and 2 Syllogism & Seating Arrangements practice tests that will help you sharpen your ability to understand and interpret data.
  • Figural Reasoning - This section has 11 next in series, 5 Odd One Out, 3 Analogies, 9 Matrices, and 2 other styles of practice tests that will strengthen your figural reasoning and make you better at seeing patterns and reaching conclusions.

Here you will find free online Analytical Reasoning example questions and answers that cover Numerical, Verbal, and Figural reasoning test topics.

Let’s dive in.

What is an Analytical Reasoning Test?

Analytical Reasoning Tests are time-constrained aptitude tests designed to assess the ability of a job candidate to think critically when faced with complicated work tasks. Additionally, employers want to know you can handle stress when required to work quickly in a deadline-driven environment.

That is why analytical thinking tests almost always include a time limit, requiring you to be quick on your feet and make decisions quickly.

If you are applying for a job that requires an aptitude test, it may be in the form of

  • Verbal Reasoning .
  • Numerical Reasoning .
  • Figural Reasoning.

That is why it is important to become familiar with and practice a variety of question types.

Almost every important industry employer seeks workers with good analytical reasoning skills, measured by analytical reasoning tests. They help employers find candidates who have the necessary cognitive abilities for learning quickly, adapting, and solving problems.

Professionals who take the Analytical Reasoning Tests include

  • Business analysts.
  • Data Analysts.
  • Intelligence Analysts.
  • Law School Admissions (In the form of the LSAT test).

Analytical Reasoning Tests Further Breakdown

Additionally to encountering analytical reasoning tests in your hiring process, the type of analytical reasoning test you'll take depends on the position you're applying for:

  • Numerical Reasoning  - Financial professionals often take numerical reasoning tests , including accountants, business analysts, and data scientists. It is estimated that three-quarters of Fortune 500 companies use psychometric testing that includes numerical reasoning tests in their recruitment processes.
  • Verbal Reasoning Questions - In most cases, employers or recruiters use verbal tests when hiring for roles that require strong interpersonal skills which is applicable to most jobs and managerial positions.
  • Figural Reasoning Questions are often administered in industries such as finance, engineering, and HR.

Analytical Reasoning Score

Upon completion of the test, the score will be analyzed and contrasted to these of other candidates or compared to a norm group statistic(which shows the hiring manager how well you measure up against previous candidates).

Free Analytical Reasoning Test Question and Answers

Analytical Reasoning - Word Problem Example Question

Due to an increase in taxes on electronic devices, the price of a 46” LED flat TV screen has increased to $845, which is 30% increase over the original price. What was the original price of the TV prior to the increase?

Choose the correct answer

The correct answer is $650.

In this question, 100% is the original price. A good way to tackle this type of question is by writing down the information you have in a table:

analytical reasoning sample answer 1

Calculate the missing data with the “triangle trick.” Multiply along the diagonal and then divide by the remaining number. Then, apply the above method to this question:

Analytical Reasoning Sample Questions Answer 1

To find the missing data, we then multiply the numbers connected by the diagonal (the hypotenuse) and then divide by the number located on the remaining vertex: X = (845*100)/130 = $650 .

Another approach to this type of question requires an understanding of the relation between a given percentage and the proportion it represents (and vice versa). This relation is represented by the following formula:

analytical reasoning sample answer 3

Total = the value of 100%. We can isolate the part we are interested in: Total = (Value*100)/% And insert the data: Value = ($845*100)/130 = $650 . Another way to tackle this question- is if you start with 130%, divide the number by 130 to get 1%. Then simply multiply the value you have received by 100.

Want More Practice?

Get the full analytical reasoning PrepPack™ including: timed tests, helpful tips and detailed answer explanations! start practicing to ensure your success on test day!

Numerical Reasoning - Number Series Example Question

7 | 14 | 28 | 32 | 64 | 128 | ? | ?

What would be the next number in the following series?

The correct answer is 3.

analytical reasoning problem solving

The series increases repeatedly by: x2, x2, +4.

Figural Reasoning - Next in Series Example Question

analytical reasoning problem solving

Choose the image that completes the pattern

analytical reasoning problem solving

The correct answer is E.

The logic: there are two rules in this set:

There is a shape in the top left corner of the frame and in the bottom left-hand corner alternately.

There is a shape in the top right corner of the frame and in the bottom right corner alternately.

This rule creates a wave-like pattern of shapes if you look at the sequence of frames. This rule already disqualifies answers 1 and 2.

The second rule concerns the shapes in the upper part of the frames only (the shapes in the bottom are only distracters and do not follow a distinct rule).

You can see that every two frames, the number of sides (of the shape) decreases by one (a pentagon, a square and a triangle).

When the shape is in the right side of the frame, it will be duplicated in the left side of the next frame, and be replaced in the following frame.

Therefore, the correct answer is 5, as the triangle is duplicated in the right place. Answer 4 may be distracting as a triangle is present, but not in the right place.

Verbal Reasoning True/False/Cannot Say Example Question

Nicotinia attenuata, a type of wild tobacco, is usually pollinated by hawkmoths. To lure them in, the plant opens its flowers at night and releases alluring chemicals. But pollinating hawkmoths often lay their eggs on the plants they visit and the voracious caterpillars start eating the plants. Fortunately, the plant has a back-up plan.

It stops producing its moth-attracting chemicals and starts opening its flowers during the day instead. This change of timing opens its nectar stores to a different pollinator that has no interest in eating it - the hummingbird.

A botanist first noticed the tobacco plant's partner-swapping antics by watching a population of flowers that was overrun by hawkmoth caterpillars. Nearly every plant was infested. To the botanist's surprise, around one in six flowers started opening between 6 and 10am, rather than their normal business hours of 6 and 10pm.

To see if the two trends were related, she deliberately infested plants from another population with young hawkmoth larvae. Eight days later, 35% of the flowers had started opening in the morning, compared to just 11% of plants not infested. The flowers use a cocktail of chemicals to lure in night-flying moths, but the main ingredient is benzyl acetone (BA).

A large plume gets released when the flower opens at night. It is so essential that genetically modified plants, which can't produce BA, never manage to attract any moths. Nonetheless, the flowers that opened in the morning never produced any BA.

Caterpillars are born of the hawkmoth's eggs.

The correct answer is True.

It logically follows that the caterpillars mentioned are hatched from the hawkmoth's eggs.

How to Improve Your Analytical Reasoning Skills
  • Practice Makes Perfect - Analytical skills tests evaluate skills that you may not use on a daily basis. It is highly recommended that you practice for your analytical reasoning test so that you will become familiar with the format of the test and the type of questions on it.
  • Focus on an Analytical Skill Relevant to Your Needs – If you know the specific skill you are going to be asked about, get professional! You can use our logical reasoning PrepPack, or our numerical reasoning PrepPack , which will enable you to focus only on the relevant material for the exam.
  • Practice Various Skills – Employers often use unique questions to analyze and evaluate your analytical thinking ability. Fortunately, our comprehensive PrepPack includes verbal, numerical, number series, and math word problems, that will ensure full preparation for any analytical test.
  • Focusing on Your Weak Spots - A guaranteed method to improve your score, even if your test is tomorrow. Aim to take a few practice tests in advance of your real test to identify your strong and weak points. Knowing this will allow you to strengthen your weaknesses just enough to secure the score you need to get the job.
  • Try Different Strategies for Approaching Analytical Reasoning Tests - For example, some test takers prefer to answer the easier questions first, leaving them with more time to approach the more difficult questions. You may also find it helpful to skip the most difficult questions and only come back to them if you have time.
  • Master Shortcut Techniques  - Shortcut techniques like mathematical tricks could help reduce the time required for a solution by half. That leaves you more time for difficult questions and gives you a huge advantage over the other candidates.
  • Build your mathematical skills – Analytical tests are widely based on mathematical skills, so if you haven’t seen an algebra question since high school - it’s time for a refresher.

Why Do Employers Use Analytical Reasoning Tests in Their Recruitment Process?

Analytical reasoning tests are often administered by employers or recruiters before the interview stage, allowing them to select candidates objectively upon merit. As a result, the test serves as a filter, enabling employers to meet the most qualified candidates and not waste valuable time.

What are analytical LSAT Reasoning Questions?

LSAT Reasoning (AR) Questions test your ability to analyze a collection of facts and principles and determine if they are true. Each AR question is based on a single passage.

Is Analytical Reasoning a Soft Skill or A Hard Skill?

The majority of analytical skills are soft, but there are also hard skills that can help you become better at analyzing. These include data analysis, data gathering, inventiveness, and the ability to communicate.

Links that May Be Helpful

  • Free Analytical Reasoning PDF - Numerical Questions.
  • Free Analytical Reasoning PDF - Verbal Questions.
  • Free Verbal Reasoning Practice Guide.
  • Free Figural Reasoning Practice Guide .
  • Free Numerical Reasoning Practice Guide.
JobTestPrep is a leading test prep company that offers accurate practice simulations for hundreds of pre-employment tests. Since 1992, it has helped 1M+ candidates. If you have any additional questions about the Analytical Reasoning Test, feel free to send us an email , we usually reply within 24 hours.

Get 25% off all test packages.

Get 25% off all test packages!

Click below to get 25% off all test packages.

What Is Analytical Reasoning?

Analytical reasoning describes a skill based on critical thinking, where you use logic to answer complex questions. By identifying patterns in facts or rules, you can use those patterns to determine outcomes that could be or must be true.

There are several different aspects of analytical reasoning that you might need to identify to be able to draw a conclusion, such as:

Causation . Being able to identify that one event is the direct cause of another – rather than correlation, where two events happened together but were not directly related.

Conditional statements . Also known as ‘if-then’ statements, conditional statements provide a hypothesis (if) and a conclusion (then). Related to this is a converse statement, which swaps the hypothesis and the conclusion.

Trends . Using quantitative data that is both measurable and exact, you can identify when an outcome is consistently increasing or decreasing over time. This is known as a trend.

Sequences . When presented with a seemingly unrelated series of numbers, images or shapes, finding the pattern or rule that makes them a sequence will help you predict the next in the series.

analytical reasoning

What is the difference between analytical reasoning and logical reasoning?

Although analytical reasoning comes under the umbrella of logical reasoning – especially in terms of pre-employment assessments – it is slightly different in scope.

Logical reasoning is about following step-by-step logical thinking to reach a conclusion, and this can be tested in many ways, from inductive and deductive reasoning to abstract reasoning.

Analytical reasoning is about interpreting the logic needed to reach a conclusion that could be or has to be true.

In practice, both terms are used interchangeably to describe problem-solving techniques.

When might you use analytical reasoning?

When we first begin to learn in school, we use analytical reasoning in the form of pattern recognition and application to read and do maths. This application of analytical reasoning can be used for learning new skills – like a language – or for picking up new information when you start a job.

As an adult, you are likely to need analytical reasoning in the workplace. Whether that is to improve communication, help resolve conflict, or to think of solutions to problems, analytical thinking is a cornerstone of higher-level roles.

Analytical reasoning is an essential part of management roles, especially those that need critical thinking and problem solving. You might find analytical reasoning most important in careers in law or financial positions, for example. For roles like these, you could encounter analytical reasoning assessments as part of the application process.

You might also need to use analytical reasoning in your everyday life, to identify trends in your spending habits when creating a budget, or when completing a planner for your future goals.

What to expect in an analytical reasoning test

When you are faced with an analytical reasoning test, understanding what to expect will make you feel more confident.

Most analytical reasoning questions are based on a passage of information, usually in sets based on the same data.

The questions are looking for your competency in finding relationships, working with conditional statements, making inferences, and finding patterns or rules to predict the next element in the sequence.

It is important to remember that these questions are designed to be answered by most people, which means that you do not need any formal training in the advertised role, or in logic, to be successful.

what is analytical reasoning

How to improve your analytical reasoning

Whenever you are presented with some information, whether it is an academic article, a news story, or even a fact, you can practice your analytical reasoning skills. Look for relationships between key pieces of information: how they are connected and whether they directly or indirectly affect one another. Think about how information is provided, so you can get clarity on the arguments and the information. You want to be able to identify key information including the wording used. Factors and elements are an important part of the way the information is presented, as well as the argument that is being made.

How to Improve Your Analytical Reasoning

Analytical reasoning is a critical skill that enables you to understand and dissect information, identify relationships, and make informed decisions. Here are strategies to enhance your analytical reasoning:

Practice Problem-Solving : Engage in activities that require problem-solving skills. This could be anything from strategic games and puzzles to real-world problems that need innovative solutions. Each problem-solving scenario can sharpen your ability to analyze and synthesize information.

Develop Quantitative Reasoning : Many analytical reasoning challenges involve quantitative elements. Improving your ability to understand and manipulate numerical data can enhance your analytical skills. Consider practicing with graphs, charts, and data sets to interpret and draw conclusions from quantitative information.

Reflect on Your Thought Process : After completing an analytical task, take a moment to reflect on your thought process. Consider what strategies worked well, what could be improved, and any insights gained during the process.

Stay Informed and Curious : Continuously expand your knowledge base and stay curious about the world around you. The more informed you are, the more context you have for analyzing new information.

By integrating these practices into your daily routine, you can progressively improve your analytical reasoning skills, making you more adept at navigating complex information and making sound decisions.

Choose a plan and start practising

Immediate access. Cancel anytime.

  • 30 Numerical reasoning tests
  • 30 Verbal reasoning tests
  • 30 Diagrammatic reasoning tests
  • 30 Situational judgement tests
  • 34 Publisher packages e.g. Watson Glaser
  • 252 Employer packages e.g. HSBC
  • 62 Extra packages e.g Mechanical
  • Dashboard performance tracking
  • Full solutions and explanations
  • Tips, tricks, guides and resources
  • Access to free tests
  • Basic performance tracking
  • Solutions & explanations
  • Tips and resources

By using our website you agree with our Cookie Policy.

Analytical Reasoning Tests (2024 Guide)

What Is an Analytical Reasoning Test?

What are analytical reasoning tests used for, analytical reasoning test practice questions (2024), tips for preparing for an analytical reasoning test in 2024, frequently asked questions, analytical reasoning tests (2024 guide).

Updated June 12, 2023

Edward Melett

All products and services featured are independently selected by WikiJob. When you register or purchase through links on this page, we may earn a commission.

Analytical reasoning tests assess a candidate’s ability to study information and apply logic to find patterns or make inferences.

At work, people use analysis to scrutinise speech, documents, diagrams, charts and graphs, and gather the most relevant information. Those with strong analytical skills will consider how key elements within that information relate to one another, and are more likely to notice crucial patterns and details.

Analytical reasoning tests measure a candidate’s critical thinking and problem-solving skills.

Data may be presented in the form of written passages, graphs, tables or shapes.

Where questions are based on a series of images , they have much in common with inductive reasoning and non-verbal reasoning tests.

Written analytical reasoning questions assess many of the same skills as verbal reasoning tests .

Prepare for Any Job Assessment Test with JobTestPrep

Recruiters use analytical reasoning tests to evaluate inductive and deductive skills in potential employees.

Deductive reasoning is the process of reaching a logical conclusion based on one or more given statements, or premises.

Inductive reasoning involves taking specific information and making predictions based on that.

Candidates do not need any specialist knowledge for analytical reasoning tests, but they must be able to think logically and pay close attention to detail. Those who demonstrate strong analytical reasoning skills are generally highly intelligent, quick to learn and more likely to improve over time in a role.

Non-verbal reasoning tests can also be helpful in assessing international candidates, or applicants who do not have English as their first language.

Candidates applying for mid- to higher-level positions may be asked to take an analytical reasoning test as part of the selection process. Analytical skills are particularly important for jobs that involve maths and numerical reasoning.

They also extend into roles where decision-making and problem-solving are key. So companies may use these tests when recruiting for positions such as computer software engineers, financial analysts, human resources managers and office managers.

Many law firms also assess analytical reasoning skills as part of their recruitment process. And analytical reasoning tests may form part of a leadership assessment process, such as a graduate recruitment scheme.

Analytical reasoning tests use both verbal and non-verbal questions.

In inductive reasoning tests , questions usually involve a series of diagrams or pictures. The candidate must find the pattern, rule or link between each item. They can then use this knowledge to decide what comes next in the sequence.

Deductive reasoning tests are typically verbal. The candidate must read a statement, or series of statements, and then choose the logically correct answer.

Examples of both are given below, with answers and explanations.

Question 1: Which Box Is Next in the Sequence?

This is an example of a question where things move around. There are many variations on this theme.

At its most basic level, elements will move around inside a box and the candidate must understand why they are moving in a particular order. By understanding this they will be able to correctly select the image that comes next.

Analytical Reasoning Tests

Which box is next in the sequence?

If you need to prepare for a number of different employment tests and want to outsmart the competition, choose a Premium Membership from JobTestPrep . You will get access to three PrepPacks of your choice, from a database that covers all the major test providers and employers and tailored profession packs.

Question 2: Which Box Is Next in the Sequence?

Candidates may also be asked to find the relationship between a set of items. There are a number of ways that elements can have relationships with one another, for example:

  • Where they are in relation to each other
  • The number of sides that different shapes have in relation to each other
  • Numbers that incrementally increase or decrease

To solve these types of questions you need to identify the rule that governs the relationship and then apply it, as in the following example:

Analytical Reasoning Tests

In this sort of question, remember also to look for relationships between odd and even numbers.

Question 3: What Most Weakens the Argument?

The following question is an example of deductive reasoning . Here the candidate must read the passage and then come to a logically correct conclusion.

This question involves identifying an assumption . An assumption is a belief that is not explicitly stated within the text but must exist to link the argument’s evidence and conclusion. To successfully answer these types of question you must find that missing link between the evidence and conclusion and then fill it.

"If all beaches were publicly owned, we would have to rely on government funds to maintain them. It is true that more people would have access to the ocean and beaches, but at what cost? If the beaches are not cared for adequately, soon there will be nothing left worth having access to. We should consider carefully before nationalising more coastal property."

Which of the following, if true , would most weaken the argument above?

A – The public does not want additional access to beaches. B – The government is currently responsible for the maintenance of all public and private beaches. C – The public already has some access to many beaches. D – Other property has been nationalised in the past, with no complaints from the original owners of the property. E – Some privately owned beaches are not well maintained.

Analytical reasoning tests can be daunting, even for confident problem solvers. You may not have come across these types of questions before, so it is essential to take plenty of time to prepare properly. This will prevent you from panicking and ensure that you gain the highest score possible.

The following tips and techniques will help you to begin the test ready to perform your best:

Know what to expect . Employers and test publishers may use terms such as inductive reasoning, deductive reasoning, verbal reasoning or non-verbal reasoning when referring to their tests. It is worth contacting the employer or company assessing you to ask more about the test you will be sitting. Most will be helpful in clarifying the nature of the test. They might provide a few example questions too.

Practise. And then practise some more . The more tests you do, the more familiar you will become with the types of questions that may come up, and the more confident you will feel. You will also begin to develop your own strategies for solving questions. Identify which types of question you find the hardest and then focus on finding the best ways to tackle them. JobTestPrep is a useful resource for sample tests and answers.

Manage your time . It is likely that you will be under pressure to complete all the questions within an allocated time. Work out how long you have to answer each question and then stick to your schedule. Don’t waste time labouring over a question that is proving particularly difficult. Move on, and then come back to any questions you have skipped over at the end if have time to spare.

What are the main topics of analytical reasoning tests?

Analytical reasoning tests can be broken down into four key areas: inductive and deductive reasoning and verbal and non-verbal reasoning.

The purpose of the test questions is to enable employers to understand how candidates assess and interpret information.

What are the best books to prepare for analytical reasoning tests?

Many different textbooks allow you to prepare for an analytical reasoning test .

You may need to focus your study on books that specialize in the type of test you are taking.

Some books will focus on the theory behind analytical reasoning tests; others will come complete with practice test questions.

When choosing a book to purchase as a study aid, try to read some reviews to decipher if it’s the right book for you.

Top choices based upon reader feedback from Amazon include:

The Fallacy Detective: Thirty-Eight Lessons on How to Recognize Bad Reasoning (written by Nathaniel Bluedorn & Hans Bluedorn)

Powerscore LSAT Logical Reasoning Bible by David M Killoran

Which organizations and institutions use analytical reasoning tests?

Analytical reasoning tests are highly regarded by civil service, private sector employers and educational institutions because they are recognized as beneficial and insightful assessments.

These tests are commonly used for jobs including computer software professionals, financial analysts and human resources.

That is because these jobs rely on critical thinking and analytical reasoning skills.

How many questions are asked on analytical reasoning tests?

This will depend on the type of analytical reasoning test that you are being asked to take. Typically, most tests will be timed, and you can expect to have at least one minute to answer each question.

If you are taking the LSAT exam, you can expect to be asked 22-24 multiple-choice questions.

What is a good resource to use for testing and improving my logical problem-solving skills for analytical reasoning tests?

There are a few ways to prepare for an analytical reasoning test . You can buy study books from all good bookstores, but you can also make the most of practice tests online.

The advantage of free online test sites is that you can put yourself into a hypothetical testing scenario and see how you are likely to react under test conditions.

It can also help you learn how to time your test to feel confident that you have enough time to answer all of the questions.

What are the best sites to get prepared for analytical reasoning tests?

Several websites offer comprehensive resources that allow you to practice analytical reasoning tests .

One of the most well-known is JobTestPrep , a site that offers exceptional study guides, answer explanations, and practice drills to help you prepare for your analytical reasoning assessment.

As a paid-for resource, this allows you to sign up for one week, one month, or three-month subscriptions, offering value for money.

Other notable sites that offer excellent free study aids and free practice tests online include practiceapptitudetests.com and practice4me.co.uk.

What is the difference between logical, analytical, non-verbal and verbal reasoning?

Logical reasoning is a series of questions that ask the participant to answer through a logical process. To conclude, you need to take a step-by-step approach to the information provided.

In contrast, analytical reasoning is about thinking critically about information presented to you and paying close attention to detail to form a conclusion.

For example, you may be asked to spot a pattern or identify the answer based on the available information.

Non-verbal reasoning is about understanding how to analyze and interpret information based upon visual aids.

For example, this could be through shapes, diagrams or patterns. Non-verbal reasoning is important for employers to understand that you can solve problems without being limited by language barriers.

Verbal reasoning is about problem-solving using words, language, and grammar. Questions are often based on spotting word problems using a true/false premise, solving patterns, and identifying how the context of a sentence can change based upon the grammar used.

What are some examples of analytical questions?

Here is an example of a typical verbal reasoning question :

Statement: 'Many employers benefit from seasonal workforces. They like to hire students and graduates in the summer months, allowing permanent employees to take a vacation. Seasonal work is beneficial to employers because they can take advantage of qualified workforces who are close to finishing their education and have yet to secure permanent employment. Some employers provide additional training and development for their seasonal staff to encourage them to join as full-time employees after graduation. A financial incentive for employers is paying seasonal staff lower wages because they are on fixed-term contracts that may not be eligible for employee benefits.'

Q: Staff who take vacation leave can have their work covered by students.

A: True / False / Cannot say

Can I fail an analytical reasoning test?

This will depend on the type of test that you are taking. Often, there are no specific pass/fail gradings, but you will be given two distinct scores. One score will be your exact test results which will detail how many questions you got right or wrong. The other score is your percentile score. This is a comparison of your results against other test-takers.

Employers need to review your percentile score because it allows them to put your score into context. Let’s imagine that you scored 70% on your test. You may be happy with this raw score, as it indicates a high level of success. But if the rest of the test takers were scoring 90% on the test, your results suddenly don’t look as good.

In contrast, perhaps you’ve scored 80% on a test, and the rest of the test takers have scored an average of 65% – in this scenario, you would be viewed in high regard by the employer as they’ve seen your results in context.

Why do employers use Analytical Reasoning Tests?

Employers use Analytical Reasoning Tests for several reasons:

  • Assessing problem-solving skills
  • Predicting job performance
  • Objective evaluation
  • Efficient screening process
  • Identifying potential leaders
  • Enhancing diversity and inclusion

Overall, employers use Analytical Reasoning Tests to assess candidates' problem-solving skills, predict job performance, streamline the screening process, identify potential leaders and foster diversity and inclusion in their hiring practices.

How do analytical reasoning tests work?

Analytical reasoning tests, also known as logical reasoning tests or deductive reasoning tests, assess an individual's ability to analyze information, recognize patterns, draw logical conclusions and solve complex problems.

These tests are often used to evaluate a person's critical thinking skills and their capacity to make sound decisions based on logical reasoning.

You might also be interested in these other Wikijob articles:

Analytical Thinking Skills (2024 Guide)

Or explore the Aptitude Tests / Test Types sections.

Practice4Me

  • AON Hewitt G.A.T.E.
  • PI Cognitive Assessment (PLI Test)
  • Korn Ferry Leadership Assessment
  • Berke Assessment
  • Ergometrics
  • Thomas International
  • Predictive Index (PI)
  • NEO Personality Inventory
  • Leadership Assessment
  • Gallup’s CliftonStrengths
  • Sales Personality Tests
  • Personality Management Tests
  • Saville Wave
  • McQuaig Word Survey
  • Bell Personality Test
  • Myers Briggs Personality Test
  • DISC Personality Test
  • Management SJT
  • Supervisory SJT
  • Administrative SJT
  • Call Center SJT
  • Customer Service SJT
  • Firefighter SJT
  • Numerical Reasoning Tests
  • Verbal Reasoning Tests
  • Logical Reasoning Tests
  • Cognitive Ability Tests
  • Technical Aptitude Tests
  • Spatial Reasoning Tests
  • Abstract Reasoning Test
  • Deductive Reasoning Tests
  • Inductive Reasoning Tests
  • Mechanical Reasoning Tests
  • Diagrammatic Reasoning Tests
  • Fault Finding Aptitude Tests
  • Mathematical Reasoning Tests
  • Critical Thinking Tests
  • Analytical Reasoning Tests
  • Raven’s Progressive Matrices Test
  • Criteria’s CCAT
  • Matrigma Test
  • Air Traffic Controller Test
  • Administrative Assistant Exam
  • Clerical Ability Exam
  • School Secretary Tests
  • State Trooper Exam
  • Probation Officer Exam
  • FBI Entrance Exam
  • Office Assistant Exam
  • Clerk Typist Test
  • Police Records Clerk Exam
  • Canada’s Public Service Exams
  • Firefighter Exams
  • Police Exams
  • Army Aptitude Tests
  • USPS Postal Exams
  • Hiring Process by Professions

Select Page

Free Analytical Reasoning Test Questions Practice – 2024

Aptitude Written Exams

What Is Analytical Reasoning?

Analytical reasoning, also known as logical reasoning , is a problem-solving method that focuses on identifying patterns and using logic to fill in missing pieces. This form of reasoning is slightly more detached from inferences and opinions, and places great emphasis on factual evidence. Analytical reasoning can be further simplified into two categories:

  • Deductive Reasoning Deductive reasoning, sometimes called top-down logic, is the formation of a specific conclusion from a general premise or, in some cases, multiple hypotheses. Deductive reasoning is best for situations with multiple variables that must be accounted for and handled.
  • Inductive Reasoning Inductive reasoning , also known as bottom-up logic, is the direct opposite of deductive reasoning as it forms plausible conclusions from a specific observation. Inductive reasoning works best when using context and practical intelligence to resolve an issue.
  • Numerical Reasoning Numerical reasoning is the ability to apply logic to problems that include data, charts and graphs. Someone who has strong numerical reasoning skills is proficient in basic mathematical functions, statistical interpretation, and algorithms to name a few.
  • Abstract Reasoning Abstract reasoning is the ability to identify patterns, extract main ideas, and analyze information. Abstract reasoning is an important factor of problem solving and decision making. This form of reasoning is complex and includes various aspects.

What Is an Analytical Reasoning Test?

Analytic reasoning tests are utilized by companies that wish to evaluate a candidate’s critical reasoning skills and their ability to pay attention to detail. This test is commonly administered for job seekers applying for mid- or high-level jobs that require a strength in mathematics. Management and human resource positions are also asked to partake in an analytical reasoning test so employers can measure their problem-solving capabilities. Depending on the role and company, participants may see one of three versions of the analytical reasoning assessment:

  • Data Most common in the financial industry, the data heavy version of the analytical reasoning test supplies applicants with graphs, charts and tables with subsequent questions. The math aspect of the data version is usually simple, so the problem-solving method can be highlighted and evaluated.
  • Written The written portion is used for leadership and management roles. Provided a passage, candidates must discern patterns and main ideas using the context and answer the multiple-choice questions that follow.
  • Images The analytical reasoning test that focuses on image-related questions slightly resembles the non-verbal reasoning test. The applicant is supplied puzzle-like questions containing shape sequences and objects and must find the missing piece.

How to Prepare for an Analytical Reasoning Test?

Analytical reasoning test can be difficult to study for because it examines your raw critical thinking skills. If you are looking to optimize and improve your score, the best way to prepare is running through sample questions or completing online practice tests. By doing this, you can become familiar with the idea of the content and the learn how to pace yourself under the time constraint. Due to the acumen required for each question, preparing for the time limit is essential because many candidates find themselves skipping and guessing at the questions rather than dedicating time and effort to them. It is also helpful to understand the qualities employers are looking for by administering these tests, that way you can narrow your preparation.

Analytical Skills Test Sample Questions

  • The facts prove the conclusion.
  • The facts disprove the conclusion.
  • Cannot say based on the information provided.

analytical reasoning job question

Aptitude Tests

  • Aptitude Tests Guide
  • Numerical Reasoning Test
  • Verbal Reasoning Test
  • Cognitive Ability Test
  • Critical Thinking Test
  • Logical Reasoning Test
  • Spatial Reasoning Test
  • Technical Aptitude Test
  • Inductive Reasoning Test
  • Analytical Reasoning Test
  • Deductive Reasoning Test
  • Mechanical Reasoning Test
  • Non-Verbal Reasoning Tests
  • Diagrammatic Reasoning Test
  • Concentration Assessment Test
  • Finance Reasoning Aptitude Test
  • Fault Finding (Fault Diagnosis) Test
  • Senior Management Aptitude Tests
  • Error Checking Tests
  • In-Basket Exercise

Register now

How it works

Transform your enterprise with the scalable mindsets, skills, & behavior change that drive performance.

Explore how BetterUp connects to your core business systems.

We pair AI with the latest in human-centered coaching to drive powerful, lasting learning and behavior change.

Build leaders that accelerate team performance and engagement.

Unlock performance potential at scale with AI-powered curated growth journeys.

Build resilience, well-being and agility to drive performance across your entire enterprise.

Transform your business, starting with your sales leaders.

Unlock business impact from the top with executive coaching.

Foster a culture of inclusion and belonging.

Accelerate the performance and potential of your agencies and employees.

See how innovative organizations use BetterUp to build a thriving workforce.

Discover how BetterUp measurably impacts key business outcomes for organizations like yours.

A demo is the first step to transforming your business. Meet with us to develop a plan for attaining your goals.

Request a demo

  • What is coaching?

Learn how 1:1 coaching works, who its for, and if it's right for you.

Accelerate your personal and professional growth with the expert guidance of a BetterUp Coach.

Types of Coaching

Navigate career transitions, accelerate your professional growth, and achieve your career goals with expert coaching.

Enhance your communication skills for better personal and professional relationships, with tailored coaching that focuses on your needs.

Find balance, resilience, and well-being in all areas of your life with holistic coaching designed to empower you.

Discover your perfect match : Take our 5-minute assessment and let us pair you with one of our top Coaches tailored just for you.

Find your Coach

Research, expert insights, and resources to develop courageous leaders within your organization.

Best practices, research, and tools to fuel individual and business growth.

View on-demand BetterUp events and learn about upcoming live discussions.

The latest insights and ideas for building a high-performing workplace.

  • BetterUp Briefing

The online magazine that helps you understand tomorrow's workforce trends, today.

Innovative research featured in peer-reviewed journals, press, and more.

Founded in 2022 to deepen the understanding of the intersection of well-being, purpose, and performance

We're on a mission to help everyone live with clarity, purpose, and passion.

Join us and create impactful change.

Read the buzz about BetterUp.

Meet the leadership that's passionate about empowering your workforce.

Find your Coach

For Business

For Individuals

What are analytical skills? Examples and how to level up

two-men-looking-at-analytics-analytical-skills

Jump to section

What are analytical skills?

Why are analytical skills important, 9 analytical skills examples, how to improve analytical skills, how to show analytical skills in a job application, the benefits of an analytical mind.

With market forecasts, performance metrics, and KPIs, work throws a lot of information at you. 

If you want to stay ahead of the curve, not only do you have to make sense of the data that comes your way — you need to put it to good use. And that requires analytical skills.

You likely use analytical thinking skills every day without realizing it, like when you solve complex problems or prioritize tasks . But understanding the meaning of analysis skills in a job description, why you should include them in your professional development plan, and what makes them vital to every position can help advance your career.

Analytical skills, or analysis skills, are the ones you use to research and interpret information. Although you might associate them with data analysis, they help you think critically about an issue, make decisions , and solve problems in any context. That means anytime you’re brainstorming for a solution or reviewing a project that didn’t go smoothly, you’re analyzing information to find a conclusion. With so many applications, they’re relevant for nearly every job, making them a must-have on your resume.

Analytical skills help you think objectively about information and come to informed conclusions. Positions that consider these skills the most essential qualification grew by 92% between 1980 and 2018 , which shows just how in-demand they are. And according to Statista, global data creation will grow to more than 180 zettabytes by 2025 — a number with 21 zeros. That data informs every industry, from tech to marketing.

Even if you don’t interact with statistics and data on the job, you still need analytical skills to be successful. They’re incredibly valuable because:

  • They’re transferable: You can use analysis skills in a variety of professional contexts and in different areas of your life, like making major decisions as a family or setting better long-term personal goals.
  • They build agility: Whether you’re starting a new position or experiencing a workplace shift, analysis helps you understand and adapt quickly to changing conditions. 
  • They foster innovation: Analytical skills can help you troubleshoot processes or operational improvements that increase productivity and profitability.
  • They make you an attractive candidate: Companies are always looking for future leaders who can build company value. Developing a strong analytical skill set shows potential employers that you’re an intelligent, growth-oriented candidate.

If the thought of evaluating data feels unintuitive, or if math and statistics aren’t your strong suits, don’t stress. Many examples of analytical thinking skills don’t involve numbers. You can build your logic and analysis abilities through a variety of capacities, such as:

1. Brainstorming

Using the information in front of you to generate new ideas is a valuable transferable skill that helps you innovate at work . Developing your brainstorming techniques leads to better collaboration and organizational growth, whether you’re thinking of team bonding activities or troubleshooting a project roadblock. Related skills include benchmarking, diagnosis, and judgment to adequately assess situations and find solutions.

2. Communication

Becoming proficient at analysis is one thing, but you should also know how to communicate your findings to your audience — especially if they don’t have the same context or experience as you. Strong communication skills like public speaking , active listening , and storytelling can help you strategize the best ways to get the message out and collaborate with your team . And thinking critically about how to approach difficult conversations or persuade someone to see your point relies on these skills. 

3. Creativity

You might not associate analysis with your creativity skills, but if you want to find an innovative approach to an age-old problem, you’ll need to combine data with creative thinking . This can help you establish effective metrics, spot trends others miss, and see why the most obvious answer to a problem isn’t always the best. Skills that can help you to think outside the box include strategic planning, collaboration, and integration.

desk-with-different-work-elements-analytical-skills

4. Critical thinking

Processing information and determining what’s valuable requires critical thinking skills . They help you avoid the cognitive biases that prevent innovation and growth, allowing you to see things as they really are and understand their relevance. Essential skills to turn yourself into a critical thinker are comparative analysis, business intelligence, and inference.

5. Data analytics

When it comes to large volumes of information, a skilled analytical thinker can sort the beneficial from the irrelevant. Data skills give you the tools to identify trends and patterns and visualize outcomes before they impact an organization or project’s performance. Some of the most common skills you can develop are prescriptive analysis and return on investment (ROI) analysis.

6. Forecasting

Predicting future business, market, and cultural trends better positions your organization to take advantage of new opportunities or prepare for downturns. Business forecasting requires a mix of research skills and predictive abilities, like statistical analysis and data visualization, and the ability to present your findings clearly.

7. Logical reasoning

Becoming a logical thinker means learning to observe and analyze situations to draw rational and objective conclusions. With logic, you can evaluate available facts, identify patterns or correlations, and use them to improve decision-making outcomes. If you’re looking to improve in this area, consider developing inductive and deductive reasoning skills.

8. Problem-solving

Problem-solving appears in all facets of your life — not just work. Effectively finding solutions to any issue takes analysis and logic, and you also need to take initiative with clear action plans . To improve your problem-solving skills , invest in developing visualization , collaboration, and goal-setting skills.

9. Research

Knowing how to locate information is just as valuable as understanding what to do with it. With research skills, you’ll recognize and collect data relevant to the problem you’re trying to solve or the initiative you’re trying to start. You can improve these skills by learning about data collection techniques, accuracy evaluation, and metrics.

handing-over-papers-analytical-skills

You don’t need to earn a degree in data science to develop these skills. All it takes is time, practice, and commitment. Everything from work experience to hobbies can help you learn new things and make progress. Try a few of these ideas and stick with the ones you enjoy:

1. Document your skill set

The next time you encounter a problem and need to find solutions, take time to assess your process. Ask yourself:

  • What facts are you considering?
  • Do you ask for help or research on your own? What are your sources of advice?
  • What does your brainstorming process look like?
  • How do you make and execute a final decision?
  • Do you reflect on the outcomes of your choices to identify lessons and opportunities for improvement?
  • Are there any mistakes you find yourself making repeatedly?
  • What problems do you constantly solve easily? 

These questions can give insight into your analytical strengths and weaknesses and point you toward opportunities for growth.

2. Take courses

Many online and in-person courses can expand your logical thinking and analysis skills. They don’t necessarily have to involve information sciences. Just choose something that trains your brain and fills in your skills gaps . 

Consider studying philosophy to learn how to develop your arguments or public speaking to better communicate the results of your research. You could also work on your hard skills with tools like Microsoft Excel and learn how to crunch numbers effectively. Whatever you choose, you can explore different online courses or certification programs to upskill. 

3. Analyze everything

Spend time consciously and critically evaluating everything — your surroundings, work processes, and even the way you interact with others. Integrating analysis into your day-to-day helps you practice. The analytical part of your brain is like a muscle, and the more you use it, the stronger it’ll become. 

After reading a book, listening to a podcast, or watching a movie, take some time to analyze what you watched. What were the messages? What did you learn? How was it delivered? Taking this approach to media will help you apply it to other scenarios in your life. 

If you’re giving a presentation at work or helping your team upskill , use the opportunity to flex the analytical side of your brain. For effective teaching, you’ll need to process and analyze the topic thoroughly, which requires skills like logic and communication. You also have to analyze others’ learning styles and adjust your teachings to match them. 

5. Play games

Spend your commute or weekends working on your skills in a way you enjoy. Try doing logic games like Sudoku and crossword puzzles during work breaks to foster critical thinking. And you can also integrate analytical skills into your existing hobbies. According to researcher Rakesh Ghildiyal, even team sports like soccer or hockey will stretch your capacity for analysis and strategic thinking . 

6. Ask questions

According to a study in Tr ends in Cognitive Sciences, being curious improves cognitive function , helping you develop problem-solving skills, retention, and memory. Start speaking up in meetings and questioning the why and how of different decisions around you. You’ll think more critically and even help your team find breakthrough solutions they otherwise wouldn’t.

7.Seek advice

If you’re unsure what analytical skills you need to develop, try asking your manager or colleagues for feedback . Their outside perspective offers insight you might not find within, like patterns in. And if you’re looking for more consistent guidance, talking to a coach can help you spot weaknesses and set goals for the long term.

8. Pursue opportunities

Speak to your manager about participating in special projects that could help you develop and flex your skills. If you’d like to learn about SEO or market research, ask to shadow someone in the ecommerce or marketing departments. If you’re interested in business forecasting, talk to the data analysis team. Taking initiative demonstrates a desire to learn and shows leadership that you’re eager to grow. 

group-of-analytic-papers-analytical-skills

Shining a spotlight on your analytical skills can help you at any stage of your job search. But since they take many forms, it’s best to be specific and show potential employers exactly why and how they make you a better candidate. Here are a few ways you can showcase them to the fullest:

1. In your cover letter

Your cover letter crafts a narrative around your skills and work experience. Use it to tell a story about how you put your analytical skills to use to solve a problem or improve workflow. Make sure to include concrete details to explain your thought process and solution — just keep it concise. Relate it back to the job description to show the hiring manager or recruiter you have the qualifications necessary to succeed.

2. On your resume

Depending on the type of resume you’re writing, there are many opportunities to convey your analytical skills to a potential employer. You could include them in sections like: 

  • Professional summary: If you decide to include a summary, describe yourself as an analytical person or a problem-solver, whichever relates best to the job posting. 
  • Work experience: Describe all the ways your skill for analysis has helped you perform or go above and beyond your responsibilities. Be sure to include specific details about challenges and outcomes related to the role you’re applying for to show how you use those skills. 
  • Skills section: If your resume has a skill-specific section, itemize the analytical abilities you’ve developed over your career. These can include hard analytical skills like predictive modeling as well as interpersonal skills like communication.

3. During a job interview

As part of your interview preparation , list your professional accomplishments and the skills that helped along the way, such as problem-solving, data literacy, or strategic thinking. Then, pull them together into confident answers to common interview questions using the STAR method to give the interviewer a holistic picture of your skill set.

Developing analytical skills isn’t only helpful in the workplace. It’s essential to life. You’ll use them daily whenever you read the news, make a major purchase, or interact with others. Learning to critically evaluate information can benefit your relationships and help you feel more confident in your decisions, whether you’re weighing your personal budget or making a big career change .

Invest in your career

Get your promotion. Make your career change. Build the future you dream about. And do it faster with a world-class BetterUp Coach by your side.

Elizabeth Perry, ACC

Elizabeth Perry is a Coach Community Manager at BetterUp. She uses strategic engagement strategies to cultivate a learning community across a global network of Coaches through in-person and virtual experiences, technology-enabled platforms, and strategic coaching industry partnerships. With over 3 years of coaching experience and a certification in transformative leadership and life coaching from Sofia University, Elizabeth leverages transpersonal psychology expertise to help coaches and clients gain awareness of their behavioral and thought patterns, discover their purpose and passions, and elevate their potential. She is a lifelong student of psychology, personal growth, and human potential as well as an ICF-certified ACC transpersonal life and leadership Coach.

20 examples of development opportunities that can level up your career

Create a networking plan in 7 easy steps, professional development is for everyone (we’re looking at you), are you being passed over for a promotion here’s what to do, discover how to get noticed by upper management at work, how to pursue jobs versus careers to achieve different goals, a guide on how to find the right mentor for your career, 8 examples for setting professional development goals at work, your guide to individual development plans (with examples), similar articles, how to develop critical thinking skills, why we're facing a crisis of imagination, and how to overcome it, what business acumen is and 9 ways to develop it, 10 essential business skills that make an impact on your career, use a personal swot analysis to discover your strengths and weaknesses, 17 essential transferable skills to boost your job search, critical thinking is the one skillset you can't afford not to master, what are metacognitive skills examples in everyday life, ready for a fresh start 7 best jobs for a career change, stay connected with betterup, get our newsletter, event invites, plus product insights and research..

3100 E 5th Street, Suite 350 Austin, TX 78702

  • Platform Overview
  • Integrations
  • Powered by AI
  • BetterUp Lead™
  • BetterUp Manage™
  • BetterUp Care®
  • Sales Performance
  • Diversity & Inclusion
  • Case Studies
  • Why BetterUp?
  • About Coaching
  • Find your Coach
  • Career Coaching
  • Communication Coaching
  • Life Coaching
  • News and Press
  • Leadership Team
  • Become a BetterUp Coach
  • BetterUp Labs
  • Center for Purpose & Performance
  • Leadership Training
  • Business Coaching
  • Contact Support
  • Contact Sales
  • Privacy Policy
  • Acceptable Use Policy
  • Trust & Security
  • Cookie Preferences
  • IBPS RRB Exam 2023 - Free Course
  • Current Affairs
  • General Knowledge
  • SSC CGL Pre.Yrs.Papers
  • SSC CGL Practice Papers
  • SBI Clerk PYQ
  • IBPS PO PYQ
  • IBPS Clerk PYQ
  • SBI PO Practice Paper

Analytical Reasoning Questions: Concepts, Syllabus Topics, Examples

  • Syllogism: Verbal Reasoning Questions and Answers
  • Analogy Reasoning Questions and Answers
  • Grouping of Figures Reasoning Questions (Concepts and Practice set)
  • Picture Analogies Questions - Non Verbal Reasoning
  • Statement and Argument-Analytical Reasoning
  • Series Completion - Verbal Reasoning Questions and Answers
  • Assertion and Reason Questions- Logical Reasoning
  • SSC CGL Prelims Reasoning Question Paper 2020
  • Logical Reasoning Questions and Answers
  • Course of Action: Logical Reasoning Questions
  • Reasoning Syllabus for IBPS PO
  • Statement and Conclusion Logical Reasoning
  • Making Judgements: Reasoning Questions
  • Letter Series Reasoning Solved Question
  • 90 Most Asked Aptitude and Reasoning Questions in Interviews
  • Analytical Writing Section in GRE General
  • SAT Exam Maths Syllabus 2024: Important Topics & Preparation Tips
  • Analytics Quotient Interview Experience (On campus)
  • Analytics Quotient Interview Experience (On Campus)

Analytical Reasoning is one of the most essential parts of the SSC CGL exam. The examiner’s intention behind asking the analytically based questions in any exam is to judge the candidate’s ability to visualize, analyze, and their creativity. 

In this article, we have covered everything you need to know about the basics of Analytical Reasoning and how you can solve those reasoning questions. we have discussed the concepts Syllabus, Reasoning Examples along with syllabus topics for many government exams and many more.

So let’s dive right in.

Analytical-Reasoning-Questions-Concepts-Syllabus-Topics-Examples

Analytical Reasoning Questions

Table of Content

Analytical Reasoning Basic Concepts

Common types of analytical reasoning, analytical reasoning syllabus, analytical reasoning questions solving tips, analytical reasoning questions – examples.

In simple terms Analytical Reasoning refers to the ability to examine and understand information by breaking down information to its fundamental components and reasons up from there. It is more like first principle thinking Which involves analyzing the relationships between different elements, identifying patterns, and using logical thinking to solve problems.

It can be used in our everyday like in various contexts, such as problem-solving, decision-making, and critical thinking. Analytical reasoning helps individuals make informed choices and understand the underlying principles or logic in a given situation. Examples of the Analytical Reasoning includes the following:

Puzzle Solving Data Interpretation Logical Reasoning Problems Critical Thinking Scenarios Scientific Method Pattern Recognition Legal Reasoning Problem-Solving in Everyday Situations

The following are some common types of Analytical Reasoning with Examples:

The following is the syllabus for the Analytical Reasoning:

Solving Analytical Reasonging problems not only make you good at solving questions for exams but also it teaches you how to think critically in daily life. Here’s the 5 tips that will help you in solving the analytical reasoning questions in efficient way.

1. Understand the Question

Understand the question very deeply and don’t jump right in without understanding the details and any specific instructions giving in question.

2. Identify Key Information

Identifiy the the terms that will lead in solving the problems. Look for relationships, conditions, and constraints that will guide the analysis.

3. Visualize the Information

Use diagrams, charts, or tables to visualize the information. These will help you in Creating visual representation and building relation ship in order to solve the problems.

4. Break Down Complex Information

Break Down the complex problems and solve indivusually.

5. Work Systematically

Approach the question in styp by step Process while solving the analytical reasoning question

6. Use Logic and Deduction

It is like when certain things are true how it impact in the scenario.

7. Practice Regularly

Practice practice and Practice! It will help you solving problems subconciously and as a result you will be able to solve the Analytical reasoning question faster in the Exam.

1. Find the number of maximum possible triangles in the below figure. 

Analytical Reasoning: maximum possible triangles

maximum possible triangles

Options: A. 10 B. 19 C. 23 D. 21

Analytical Reasoning: maximum possible triangles  solution

maximum possible triangles solutions

Number of direct/ simple triangles = 10 Number of triangles composed of two points each = 7 Number of triangles composed of four points each = 4 Total number of triangles = 10+ 7 + 4 = 21

2. Determine how many straight lines are necessary to create the given figure.

Analytical reasoning: straight lines in figures

Options: A. 9 B. 11 C. 15 D. 16

Required horizontal lines are DE, FH, IL and BC i.e. 4 Required slanting lines are AC, DO, FN, IM, AB, EM and HN i.e. 7. Total number of required lines = 11

3. Count the number of rectangles in the below figure.

number of rectangles in figures

Option: A. 19 B. 16 C. 30 D. 34

Analytical Reasoning: Triangles in figures solution

Simple rectangles = 8 (VWON, OBAN,XYQP,PQCB, CRSD, RZA1S,DTUA,B1C1UT) Rectangles combining 2 Components = 8 (ABJL,BCKI,JCDL,KDAI,EFCA,BFGD,CGHA,DHEB) Squares combining 2 components = 4 (BJMI, CKMJ , DLMK and AIML). Squares combining 3 components = 4( EBMA, BFCM, MCGD and AMDH). Squares combining 4 components = 4( VWBA, XYCB, ZA1DC and B1C1AD ). Squares combining 7 components = 4( NOJL, PQKI, RSLJ and TULK). Squares combining 8 components = 1 (ABCD). Squares combining 12 components = 1 (EFGH). Hence, total number of rectangles = 4 + 4 + 4 + 4 + 1 + 1 +8+8 = 34

4. Count the number of squares and triangles in the given figure. 

Analytical Reasoning Question

A. 28 triangles, 10 squares B. 28 triangles, 8 squares C. 32 triangles, 10 squares D. 32 triangles, 8 squares

Analytical Reasoning Solutions

Triangle : The eight most basic triangles are IJQ, JKQ, KLQ, LMQ, MNQ, NOQ, OPQ, and PIQ. There are 12 triangles who has 2 compoents = ABQ, BCQ, CDQ, DEQ, EFQ, FGQ, GHQ, HAQ, IKQ, KMQ, MOQ, and OIQ. There are 12 triangles who has 4 compoents = ACQ, CEQ, EGQ, GAQ, IKM, KMO, MOI and OIK. There are 8 triangles who has 8 compoents = ACE, CEG, EGA and GAC ∴ Total number of triangles in the figure : 8 + 12 + 8 + 4 = 32. Squares : Four squares consist of two components each, which are IJQP, JKLQ, QLMN, and PQNO. Similarly, there are four squares composed of four components each, which are ABQH, BCDQ, QDEF, and HQFG. There is only one square consisting of eight components, which is IKMO. Moreover, there is only one square made up of sixteen components, which is ACEG. The figure contains a total of 10 squares. Hence, option C is correct.

Q 5. Calculate a number of triangles.

 number of triangles - No of triangles

Option. A. 8 B. 7 C. 6 D. 5

Analytical Reasoning solution

Question 6. Calculate the number of quadrilaterals in the below figure.

Analytical Reasoning Solution

Options: (A) 15 (B) 18 (C) 19 (D) 21

Analytical Reasoning Solution

Quadrilateral: Any closed shape which has 4 sides is called as quadrilateral. Now, we have to count 4 sided closed shape. No. of square=5 No. of rectangle=4 Shape with 4 sides= 9 Totall =18

Q 7. Calculate a number of rhombus in the below figure.

Analytical-reasoning: number of rohmbus

Question 8: Count number of squares.

Simple squares: 10 Combined squares: 6 (Note: Don’t count the center Square twice)

Count number of quares

Total : 16.

Question 8: Count sum of triangles and square.

Count sum of triangles and square

Triangle = 12+7+4 =23

Counts of triangles solution

Square= 6+3=9

Counts of triangles solution

Answer is 32

Question 9: Count parallogram in the below figure.

Count Parallogram Questions

Remember parallogram is trapezium, but vice versa is not true.

 Count parallogram: solutions

Conclusion:

In conclusion, Analytical Reasoning is an essential skill for the SSC CGL exam. Candidates should develop their observation and visual skills, practice regularly, and manage their time effectively to excel in this section. With the right approach and dedicated effort, anyone can improve their Non-Verbal Reasoning skills and score well in the SSC CGL exam. 

All the Best!!!

FAQs on Analytical Reasoning Questions

What is the nature of analytic reasoning.

Analytical reasoning based questions judge a person’s ablity to identify patterns and visualize them. 

What is the process of preparation for analytical reasoning?

It is not something, which one can study and learn, you need to practise questions thoroughly and practise visualizing the patterns. 

How many questions are on analytical reasoning?

It differs from one exam to other, any number of questions can be asked. 

Is analytical reasoning the same as logical?

In analytical reasoning based question one needs to use their analytical skills where as in logical questions one needs to use mental ability.

Please Login to comment...

Similar reads.

  • Analytical Mathematical Puzzles
  • Banking Reasoning
  • SSC Reasoning
  • SSC/Banking

Improve your Coding Skills with Practice

 alt=

What kind of Experience do you want to share?

Radford University

Center for Innovation and Analytics

Departments

  • Academic Affairs
  • Audit and Advisory Services
  • Finance and Administration
  • Human Resources
  • Information Technology
  • Office of the President
  • Student Affairs
  • University Advancement
  • University Relations
  • Other Offices and Departments
  • About the Center for Innovation and Analytics
  • Areas of Growth in Analytics
  • Analytics Career Preparation
  • Microsoft Office Specialist Certifications
  • Executives in Residence in Analytics
  • Success Stories
  • Analytics Events
  • SAS Joint Graduate Certificate in Business Analytics
  • Analytics Resources
  • Online SAS Joint Graduate Certificate in Business Analytics Certificate
  • The Background to Support the Center
  • What the Center Provides
  • Skills Required by Employers
  • Director's Bio

P.O. Box 6953 Radford, VA 24142 Kyle Hall Suite 231 540.831.5513 cia@radford.edu cia-analytics@radford.edu cia-innovation@radford.edu

Dr. Wil Stanton, Director wstanton@radford.edu cia-analytics@radford.edu

Vicki Perkins, Administrative Assistant vperkins1@radford.edu

Problem Solving, Critical Thinking, and Analytical Reasoning Skills Sought by Employers

In this section:

Problem Solving

  • Critical Thinking

Analytical Reasoning

View the content on this page in a Word document.

Critical thinking, analytical reasoning, and problem-solving skills are required to perform well on tasks expected by employers. 1 Having good problem-solving and critical thinking skills can make a major difference in a person’s career. 2

Every day, from an entry-level employee to the Chairman of the Board, problems need to be resolved. Whether solving a problem for a client (internal or external), supporting those who are solving problems, or discovering new problems to solve, the challenges faced may be simple/complex or easy/difficult.

A fundamental component of every manager's role is solving problems. So, helping students become a confident problem solver is critical to their success; and confidence comes from possessing an efficient and practiced problem-solving process.

Employers want employees with well-founded skills in these areas, so they ask four questions when assessing a job candidate 3 :

  • Evaluation of information: How well does the applicant assess the quality and relevance of information?
  • Analysis and Synthesis of information: How well does the applicant analyze and synthesize data and information?
  • Drawing conclusions: How well does the applicant form a conclusion from their analysis?
  • Acknowledging alternative explanations/viewpoints: How well does the applicant consider other options and acknowledge that their answer is not the only perspective?

When an employer says they want employees who are good at solving complex problems, they are saying they want employees possessing the following skills:

  • Analytical Thinking — A person who can use logic and critical thinking to analyze a situation.
  • Critical Thinking – A person who makes reasoned judgments that are logical and well thought out.
  • Initiative — A person who will step up and take action without being asked. A person who looks for opportunities to make a difference.
  • Creativity — A person who is an original thinker and have the ability to go beyond traditional approaches.
  • Resourcefulness — A person who will adapt to new/difficult situations and devise ways to overcome obstacles.
  • Determination — A person who is persistent and does not give up easily.
  • Results-Oriented — A person whose focus is on getting the problem solved.

Two of the major components of problem-solving skills are critical thinking and analytical reasoning.  These two skills are at the top of skills required of applicants by employers.

- Return to top of page -

Critical Thinking 4

“Mentions of critical thinking in job postings have doubled since 2009, according to an analysis by career-search site Indeed.com.” 5 Making logical and reasoned judgments that are well thought out is at the core of critical thinking. Using critical thinking an individual will not automatically accept information or conclusions drawn from to be factual, valid, true, applicable or correct. “When students are taught how to use critical thinking to tap into their creativity to solve problems, they are more successful than other students when they enter management-training programs in large corporations.” 6

A strong applicant should question and want to make evidence-based decisions. Employers want employees who say things such as: “Is that a fact or just an opinion? Is this conclusion based on data or gut feel?” and “If you had additional data could there be alternative possibilities?” Employers seek employees who possess the skills and abilities to conceptualize, apply, analyze, synthesize, and evaluate information to reach an answer or conclusion.

Employers require critical thinking in employees because it increases the probability of a positive business outcome. Employers want employees whose thinking is intentional, purposeful, reasoned, and goal directed.

Recruiters say they want applicants with problem-solving and critical thinking skills. They “encourage applicants to prepare stories to illustrate their critical-thinking prowess, detailing, for example, the steps a club president took to improve attendance at weekly meetings.” 7

Employers want students to possess analytical reasoning/thinking skills — meaning they want to hire someone who is good at breaking down problems into smaller parts to find solutions. “The adjective, analytical, and the related verb analyze can both be traced back to the Greek verb, analyein — ‘to break up, to loosen.’ If a student is analytical, you are good at taking a problem or task and breaking it down into smaller elements in order to solve the problem or complete the task.” 9

Analytical reasoning connotes a person's general aptitude to arrive at a logical conclusion or solution to given problems. Just as with critical thinking, analytical thinking critically examines the different parts or details of something to fully understand or explain it. Analytical thinking often requires the person to use “cause and effect, similarities and differences, trends, associations between things, inter-relationships between the parts, the sequence of events, ways to solve complex problems, steps within a process, diagraming what is happening.” 10

Analytical reasoning is the ability to look at information and discern patterns within it. “The pattern could be the structure the author of the information uses to structure an argument, or trends in a large data set. By learning methods of recognizing these patterns, individuals can pull more information out of a text or data set than someone who is not using analytical reasoning to identify deeper patterns.” 11

Employers want employees to have the aptitude to apply analytical reasoning to problems faced by the business. For instance, “a quantitative analyst can break down data into patterns to discern information, such as if a decrease in sales is part of a seasonal pattern of ups and downs or part of a greater downward trend that a business should be worried about. By learning to recognize these patterns in both numbers and written arguments, an individual gains insights into the information that someone who simply takes the information at face value will miss.” 12

Managers with excellent analytical reasoning abilities are considered good at, “evaluating problems, analyzing them from more than one angle and finding a solution that works best in the given circumstances”. 13 Businesses want managers who can apply analytical reasoning skills to meet challenges and keep a business functioning smoothly

A person with good analytical reasoning and pattern recognition skills can see trends in a problem much easier than anyone else.

Dialog Image

Testzone, best Test Series for Bank, Railways and SSC exams.

Current Affairs Mock Drill 2024

Current affairs mock drill 2023, current affairs mock drill 2022, current affairs mock drill 2021.

Telegram

Analytical Reasoning Questions with Answers

Analytical reasoning quiz 1.

Analytical reasoning, a key component of non-verbal reasoning sections competitive exams like SSC, Railway, Insurance and Banking and involves the application of analytical abilities to deduce answers without relying on specialized knowledge. This skill encompasses understanding the basic structure of relationships based on given facts and rules. At Smartkeeda, we recognize the significance of mastering analytical reasoning and offer a comprehensive set of analytical reasoning questions with answers to help you excel in this critical reasoning domain. Mastering analytical reasoning is essential for success in competitive exams, and our analytical reasoning practice questions provide a structured and effective way to hone this skill.

​​​​​​​Our analytical reasoning questions with answers sets are designed to evaluate your deductive reasoning skills. By engaging with these questions, you will gain the necessary knowledge on how to approach and answer them effectively. Our detailed explanation provided with each question enables you to understand the underlying logic, helping you grasp the intricacies of analytical reasoning questions. Additionally, the inclusion of short tricks aids in developing a strategic approach, allowing you to navigate these questions efficiently and accurately during exams. By downloading our analytical reasoning questions pdf , you can also attempt these questions offline.  By regularly engaging with these questions, you'll be well-prepared to tackle analytical reasoning question with confidence during your exams.

Frequently Asked Questions

How to Pass & Prepare for Analytical Reasoning Tests in {YEAR}?

What Is Analytical Reasoning Test?

What to expect when taking an analytical reasoning test, how to prepare for analytical reasoning tests in 2024, frequently asked questions, how to pass & prepare for analytical reasoning tests in 2024.

Updated November 10, 2023

Edward Melett

An analytical reasoning test is a way of measuring a candidate’s comprehension skills and their ability to identify key information, apply logic and find patterns.

This form of testing is used widely in recruitment, particularly when assessing candidates for training or graduate schemes.

Large organizations may use an analytical skills test in the early stages of the recruitment process to assess the aptitude of shortlisted candidates before moving onto the interview stage.

This allows recruiters to test several skills such as:

  • Attention to detail
  • Critical thinking
  • Ability to apply logic
  • Inductive reasoning
  • Deductive reasoning
  • Decision making

Analytical reasoning tests are often delivered online and completed by the candidate at a time of their choosing.

You will be given a final deadline for completing the test which, if missed, will eliminate you from the selection process.

The time allowed to complete the test is determined by the recruiter. Some apply rigid time constraints , so good time management and quick thinking is necessary.

Taking practice papers in advance can help you work out how long to spend on each question and which types of questions take more of your time.

Not all organizations apply a time limit, although they may give you an estimate of how long it will take or record the time taken.

In these cases, the testing program may present you with different questions, depending on how well you answered the previous one.

Prepare for Any Job Assessment Test with JobTestPrep

Analytical Reasoning Test Formats

Non-verbal reasoning.

Non-verbal reasoning, or numerical reasoning , is the ability to analyze graphs, tables and data, to draw conclusions and make predictions.

Although based upon a numerical format, this method of testing is not designed to assess your mathematical knowledge; it focuses on logic and reasoning skills instead.

Many job roles rely on analyzing numerical data, and if you are applying for roles in the finance, engineering, marketing or human resources industries, you are very likely to be asked to complete an analytical reasoning test containing non-verbal reasoning assessments.

Verbal Reasoning

Verbal reasoning is the process of using written information, identifying and analyzing key points, to draw conclusions.

These skills are often assessed during an analytical reasoning test by presenting a text-based question, such as an excerpt of a business report or research paper.

You will then be expected to answer questions by interpreting the information and applying logic to come to your conclusion.

Deductive vs Inductive Reasoning

Deductive reasoning is taking a set of facts and using them to make further statements of fact. So, for example:

  • Package C weighs the same as package D
  • Package F weighs twice as much as package D

Therefore, it is also a fact that the weight of two F packages is equal to the weight of four C packages.

Of course, there are many other facts that we can deduce from the initial statements, and a question in an analytical skills test will be more complex than this.

Inductive reasoning is the process of using the information you have to identify patterns and make predictions about what is likely to happen next.

For example, take this table listing the population levels of Town A:

If you were asked to estimate the town population for 2020, you would use inductive reasoning to make your prediction.

Of course, your answer isn’t fact – but it shows that you have identified patterns and used logic to make an informed prediction.

In a test, a question that is asking you to use your inductive reasoning skills will often ask “What comes next?”.

Inductive logic is valued highly in the fields of engineering, IT and science.

If you need to prepare for a number of different employment tests and want to outsmart the competition, choose a Premium Membership from JobTestPrep . You will get access to three PrepPacks of your choice, from a database that covers all the major test providers and employers and tailored profession packs.

Examples of Analytical Questions

Analytical question 1.

Here is an example of a relatively difficult analytical reasoning question.

It requires the candidate to use the statements that are presented to identify other statements that must also be true.

This question is a test of verbal and deductive reasoning skills:

From a group of seven undergraduate students (A, B, C, D, E, F, and G), four will be selected to give a presentation to the students' union. The following conditions must be met:

  • Either A or B must be selected, but A and B cannot both be selected.
  • Either E or F must be selected, but E and F cannot both be selected.
  • E cannot be selected unless C is selected.
  • G cannot be selected unless B is selected.

If we know that F is not selected to present, how many different groups of four can be made, following the above criteria?

a) One b) Two c) Three d) Four e) Five

Analytical Question 2

A common form of numerical (or non-verbal) reasoning question is to identify pattern rules and predict what comes next.

An example of this type of question is:

Analytical Reasoning Tests

Look at the top row of images. Which box comes next in the sequence?

Analytical Question 3

Non-verbal reasoning may also be assessed by asking a number series question. This involves spotting patterns in a series of numbers, using some basic mathematical principles to predict the next number.

For example:

What number comes next?

9, 15, 13, 19, 17, 23...

Step 1 . Research the Test You Are Taking

Several different companies specialize in creating analytical reasoning tests. Asking your recruiter which provider they use will allow you to research practice papers beforehand.

Step 2 . Prepare Your Tech

To save unnecessary stress on the day, make sure you have the hardware and software needed to complete the test.

Discovering that your laptop is about to run out of battery at the last minute is the last thing you need.

Step 3 . Prepare the Necessary Equipment

Make sure you have all the necessary equipment with you.

You may need a pen and paper for jotting down details and helping you work out your answers. If allowed, a calculator may also prove useful.

Step 4 . Read the Instructions Carefully

Take your time to read through the test instructions before you start answering questions.

Making a note of time allowance is important, so be sure to revisit this as you progress through the test to make sure you manage your time efficiently.

Step 5 . Leave the Hard Questions ‘Till Last

Consider answering the test questions in a different order than they are set.

It may help to go through the test, answering questions you feel confident in, before returning to the more difficult questions.

Try out this method in your practice tests beforehand to find what works for you.

Step 6 . Use Process of Elimination

Use a process of elimination when answering questions. Sometimes, it’s easier to spot the wrong answers than the right one. If you rule out enough of the wrong answers, you’ll get a head start on identifying the correct one.

Step 7 . Stay Calm

Most importantly, stay calm . When faced with a difficult question, break it down, making notes as you go.

Trust your ability – remember that all the necessary information is provided in the question, you are not being tested on any professional knowledge, just on your reasoning skills.

How do you prepare for an analytical test?

Start by getting familiar with the format of the analytical test. Know whether it's multiple-choice, essay-based or involves data analysis.

Review any material or content that may be relevant to the test.

Practice solving analytical problems from past tests or sample questions.

On the day of the test, try to stay calm and focused.

What is an example of an analytical thinking test?

An example of an analytical thinking test might be a case study or a situational analysis. You could be presented with a real-world problem or scenario and asked to analyze it, identify key issues, and propose solutions based on the information provided. The test may include written responses, multiple-choice questions, or both.

What is the purpose of an analytical thinking test?

The purpose of an analytical thinking test is to assess your ability to think critically, analyze complex situations, make informed decisions, and solve problems. Employers often use these tests to evaluate a candidate's problem-solving skills and decision-making abilities. In an academic context, analytical tests are used to assess a student's understanding of a subject and their ability to apply knowledge to solve problems.

What are the best ways to enhance your analytical abilities?

Regularly engage in activities that require critical thinking, such as puzzles, brain teasers or debates.

Reading a variety of materials, from books to news articles, can expose you to different perspectives and ideas, enhancing your analytical skills.

Work on solving problems, both simple and complex, to strengthen your ability to analyze situations and find solutions.

Stay updated in your field and keep learning, as new information and perspectives can enhance your analytical abilities.

Many corporate recruiters rely on analytical reasoning tests as part of their selection process.

This type of psychometric test is designed to assess a candidate’s general aptitude as well as their ability to apply logic and reasoning.

Results are data-driven and can provide an objective measure of a candidate’s potential to fulfill the role they are applying for.

With adequate preparation, this is an ideal opportunity for a candidate to demonstrate their logical ability and professional competence.

Scores are compared against those of other candidates and will influence progression onto the next round of the process.

In some cases, candidates will automatically be discounted if they have poor results on the reasoning test.

Therefore, it is important that you practice. Take online practice tests to ensure your skills are honed before your test.

You might also be interested in these other Psychometric Success articles:

Logical Reasoning Tests: 2024 Guide for Logic Tests

Or explore the Aptitude Tests / Test Types sections.

  • Prompt Library
  • DS/AI Trends
  • Stats Tools
  • Interview Questions
  • Generative AI
  • Machine Learning
  • Deep Learning

Analytical thinking & Reasoning: Real-life Examples

analytical thinking 1

Analytical thinking and analytical reasoning are two concepts that are often misunderstood. Many people think that they are the same thing, but this is not the case. In fact, analytical thinking and analytical reasoning are two very different things, however, related. Analytical thinking is an important aspect of analytical skills. Most of us do not realize how to use analytical thinking and often end up solving the problem incorrectly or half-heartedly. As data analysts or data scientists , it would be of utmost importance to acquire this skill well. In this blog post, we will learn these concepts with the help of some real-life examples.

Table of Contents

What’s Analytical Thinking?

Before we get into understanding what is analytical thinking, lets understand the word, analysis , which forms the word, analytical.

The word “analysis” comes from the Ancient Greek ἀνάλυσις (analysis, “ a breaking-up” or “an untying;” from ana- “ up , throughout” and lysis “a loosening “). 

From above, it can be comprehended that loosening anything or any problem up can said to be analyzing the thing or the problem. And, loosening a problem or a thing can be represented as breaking down the problem or thing into further components (sub-problems or sub-things). 

Given above, let’s understand what is analytical thinking?

Analytical thinking is the process of thinking about any topic or an issue / problem which involves breaking down a problem / issue or a topic into smaller parts in order to better understand it in a better manner. When working with a problem or an issue, analytical thinking helps find great solutions. When trying to understand a topic, analytical thinking helps understand topic and related concepts in a better manner.

Analytical thinking fits in very well with the first principles thinking in the sense that reasoning from first principles requires you to break down a thing, an idea or a problem into its most basic elements to know the problem / idea / thing well. Thus, one would need to do analytical thinking or be analytical to do first principles thinking . This can be done by asking questions such as some of the following:

  • What is the problem which needed to be solved? For example, how do we run the schools in Covid times in a safe manner? How do we increase sales? How do we achieve cost savings target in procurement? How do we reduce loss due to credit card fraud transactions?
  • Are there sub-problems which when solved can help solving the problem as a whole? For example, the covid problem can be broken down into sub-problems related to vaccination, masking, sanitization, social distancing, covid communication etc. Similarly, the challenge of achieving cost savings in procurement can be broken down into avoiding pricing deviations, increasing reuse, proper budget planning, proper inventory planning, etc. These can further be broken down into its most basic elements based on first principles thinking . 
  • Why are we trying to solve the problems and/or sub-problems? What is going to be the ultimate change which will happen as a result of solving the problem? For example, the schools opening up will prove to be good for mental and physical well-being of the students. The cost saving in procurement will result in positive impact on the bottom line of the company. 
  • What constitutes the core of the problem/sub-problems? Who can create impact and who will be impacted? For example, the school and its students, teachers, staff members, parents, etc. In first principles thinking , this is also called as efficient cause . It is important to understand this aspect as solution could also be related to changing the way the end user deal with the situations related to the problem space.
  • What processes will be impacted? Or, what processes will need to be changed? 

Here is the diagram representing the above:

analytical thinking 1

You may want to check a related post which will help you break the problem into sub-problems using first principles thinking. Here is the post titled – First principles thinking explained with examples .

Asking questions is key to analytical thinking

Asking questions holds key to the analytical thinking or breaking down the problem or thing into smaller components (sub-problems). The questions can be of different forms such as the following:

  • What do we assume about this situation?
  • What assumptions have been made here that may need to be challenged?
  • What are the underlying assumptions behind this decision?
  • Can you explain what you mean by ABC?
  • What do you think the implications of XYZ could be?
  • How does PQR compare to other similar topics?
  • What other perspectives should be considered in relation to EFG?
  • How could this decision impact our business in the long-term?
  • What would be the consequences if we implemented this solution?
  • How will our customers react to this change?
  • What unintended outcomes may result from us taking this approach?  

What’s Analytical Reasoning?

Analytical reasoning is the process of forming conclusions from given premises by applying valid inference rules while considering different interpretations or perspectives on them. It is the ability to use logic and reason to solve problems. It helps us use what we know to figure out what else might be true. This can be done by following the processes. The steps will be described using the example.

  • Set hypothesis : First and foremost, one needs to set a hypothesis. For example, walking 5 KM on daily basis reduces risk of heart attack.
  • Figure out reasons ; The reason for reduction in risk of heart attack is the reduction in cholesterol to acceptable limits, or, keeping the cholesterol well within acceptable limits.
  • State general proposition with examples : State general proposition which is universally accepted as true with the help of examples. The cholesterol level well within the universally accepted limit is found to be indicator of good health of heart. As a data scientist , one can perform hypothesis testing at this stage to sight multiple examples. You may want to check one of my related posts: Hypothesis testing explained with examples .
  • Re-instate the general proposition to the current hypothesis ; Walking 5 KM a day keeps the cholesterol level well below acceptable limits. This can be outcome of the hypothesis tests you performed in the previous step.
  • Draw the conclusion from the previously stated general proposition . Walking 5 KM on daily basis reduces the risk of heart attack.

The above reasoning process is inspired by Nyaya Syllogism from Nyaya philosophy (One of the schools of Indian Philosophy ).

Here is another simpler and common example of analytical reasoning:

  • Set hypothesis: The house is burning with fire
  • Figure out reasons: There is smoke coming out of house
  • State the general proposition with examples: When there is a smoke, there is a fire. For example, kitchen
  • Re-instate the general proposition to the current hypothesis: The house is having lot of smoke.
  • Draw the conclusion: The house is burning with fire.

Analytical Thinking Sample Use Case – Problem of Traffic

In this section, we will take a look at the approach we take to understand / analyze the problem of traffic in the city during peak hours.

To break down the problem of traffic in the city during peak hours, we can start by asking a few key questions and identifying sub-problems. Here are some possible sub-problems to consider:

  • What causes traffic during peak hours?
  • Are there specific areas or routes where traffic is particularly bad?
  • What is the impact of traffic on commuters, businesses, and the environment?
  • Are there any existing solutions or initiatives in place to address traffic during peak hours?
  • What are the trade-offs or unintended consequences of different solutions?

To further analyze these sub-problems, we can ask a range of questions. Here are some examples of assumption, clarification, and consequence questions that might be helpful:

Assumption questions:

  • What assumptions are we making about the causes of traffic during peak hours?
  • Are there any biases or preconceptions we need to be aware of when analyzing this problem?
  • Are there any assumptions we’re making about the impact of traffic on different stakeholders?
  • What assumptions are we making about the behavior of drivers and commuters during peak hours?
  • Are there any assumptions we’re making about the types of vehicles or transportation modes that contribute to traffic during peak hours?
  • What assumptions are we making about the impact of new technologies, such as autonomous vehicles or ride-sharing platforms, on traffic during peak hours?

Clarification questions:

  • What data do we have about traffic patterns and congestion during peak hours?
  • What are the specific pain points for commuters and businesses when it comes to traffic?
  • Are there any regulations or policies that are contributing to or exacerbating traffic during peak hours?
  • What are the key factors that contribute to congestion during peak hours, such as bottlenecks, intersections, or merging points?
  • How does public transportation, such as buses or trains, impact traffic during peak hours?
  • Are there any data or studies on the impact of remote work or flexible schedules on traffic during peak hours?
  • What is the impact of school start and end times on traffic during peak hours?

Consequence questions:

  • What is the ultimate goal of solving this problem?
  • What are the benefits of reducing traffic during peak hours for different stakeholders?
  • What are the costs of not addressing this problem?
  • How might solving this problem impact the environment, public health, and safety?
  • What are the long-term implications of reducing traffic during peak hours for the city’s infrastructure and economy?
  • What are the potential unintended consequences of different solutions or initiatives?
  • What are the trade-offs involved in implementing different solutions?

By breaking down the problem into sub-problems and asking these types of questions, we can gain a more comprehensive understanding of the issue and identify potential solutions to alleviate traffic during peak hours.

Presentation on Analytical Thinking

The following slides / presentation is an introduction to analytical thinking. Analytical thinking is characterized by the ability to break down complex information into smaller, more manageable parts to better understand it. The presentation covers the characteristics of analytical thinkers, the analytical thinking process / workflow, the benefits of analytical thinking and what should one do for developing and improving analytical thinking skills.

Analytical Thinking with ChatGPT / Generative AI Tools

Ever wondered how analytical thinking could take your interactions with ChatGPT and other generative AI tools to the next level? 🤔 Brace yourself for an exciting exploration of the dynamic relationship between analytical thinking and these cutting-edge AI technologies. In this section, we’ll look at some of the prompts which can help you do analytical thinking while leveraging generative AI tools such as ChatGPT.

The following are some of the example prompts to give you an idea on how could you start doing analytical thinking around a topic or a problem / issue.

The following prompt can be used with code interpreter in ChatGPT 4. You can upload a CSV file consisting of customer reviews or a feedback.

Act as a data analyst who is tasked with analyzing customer feedback and reviews to uncover insights about the reasons for the decline in sales.

Utilize analytical thinking to categorize and analyze customer sentiments, complaints, and preferences related to the company’s products or services. Look for recurring themes or issues that customers have been expressing to identify areas that require immediate attention. Leverage sentiment analysis to extract valuable insights from unstructured data sources. Based on your analysis, create a comprehensive report consisting of appropriate plots / charts highlighting the key customer pain points and propose data-backed strategies to address these concerns, ultimately revitalizing the company’s sales performance.

Analytical thinking and analytical reasoning are two very different things. In this blog post, we learned the concepts with the help of some real-life examples. Analytical thinking is about breaking down a problems or issues into smaller parts in order to better understand it and find solutions. The steps involved include asking questions such as: “What is the problem which needed to be solved?”; “Are there sub-problems that need to be solved as well?”; “Why are we trying to solve these problems? What will happen if we do successfully solve them?”, etc. Whereas, analytical reasoning includes forming conclusions from given premises by applying valid inference rules while considering various interpretations or perspectives on those premises. Please feel free to share your thoughts or suggestions or ask questions for clarifications.

Recent Posts

Ajitesh Kumar

  • Python Pickle Security Issues / Risk - May 31, 2024
  • Pricing Analytics in Banking: Strategies, Examples - May 15, 2024
  • How to Learn Effectively: A Holistic Approach - May 13, 2024

Ajitesh Kumar

Leave a reply cancel reply.

Your email address will not be published. Required fields are marked *

  • Search for:
  • Excellence Awaits: IITs, NITs & IIITs Journey

ChatGPT Prompts (250+)

  • Generate Design Ideas for App
  • Expand Feature Set of App
  • Create a User Journey Map for App
  • Generate Visual Design Ideas for App
  • Generate a List of Competitors for App
  • Python Pickle Security Issues / Risk
  • Pricing Analytics in Banking: Strategies, Examples
  • How to Learn Effectively: A Holistic Approach
  • How to Choose Right Statistical Tests: Examples
  • Data Lakehouses Fundamentals & Examples

Data Science / AI Trends

  • • Prepend any arxiv.org link with talk2 to load the paper into a responsive chat application
  • • Custom LLM and AI Agents (RAG) On Structured + Unstructured Data - AI Brain For Your Organization
  • • Guides, papers, lecture, notebooks and resources for prompt engineering
  • • Common tricks to make LLMs efficient and stable
  • • Machine learning in finance

Free Online Tools

  • Create Scatter Plots Online for your Excel Data
  • Histogram / Frequency Distribution Creation Tool
  • Online Pie Chart Maker Tool
  • Z-test vs T-test Decision Tool
  • Independent samples t-test calculator

Recent Comments

I found it very helpful. However the differences are not too understandable for me

Very Nice Explaination. Thankyiu very much,

in your case E respresent Member or Oraganization which include on e or more peers?

Such a informative post. Keep it up

Thank you....for your support. you given a good solution for me.

MockQuestions

analytical reasoning problem solving

Analytical Reasoning I

25 questions created by jacqueline fiore.

Every organization has specific requirements during its hiring process. Many professions require their candidate to complete a series of aptitude tests before moving on to the next step. One such test is the Analytical Reasoning Test, designed to assess you on various skills. The Analytical Reasoning Test assesses your logical and critical-thinking skills, analysis, and problem-solving.

Table of Contents

How to prepare for an analytical reasoning test.

Every organization has specific requirements during its hiring process. Many professions require their candidate to complete a series of aptitude tests before moving on to the next step. One such test is the Analytical Reasoning Test, designed to assess you on various skills. The Analytical Reasoning Test assesses your logical and critical-thinking skills, analysis, and problem-solving.Most Analytical Reasoning Tests are timed, ranging from 45 to 60 seconds per question, with 20 questions or more. Typically, calculators are not allowed, but you will be able to use a piece of paper and a pencil to calculate mathematical answers. Employers want to determine that you can keep calm under pressure and work quickly against the clock. Most workplaces administer Analytical Reasoning Tests online, but some may have you visit a training center to take your test.

What is an Analytical Reasoning Test?

An Analytical Reasoning Test is designed to assess skills that are used in most industries. This test measures your ability to learn quickly, adapt, and solve problems. Depending on the type of job you are applying for, the test may be designed to assess your verbal reasoning, non-verbal reasoning, inductive reasoning, and deductive reasoning.

Why do Companies Administer Analytical Reasoning Tests?

Companies that administer Analytical Reasoning Tests do so if problem-solving is a requirement for the position or if they want a good indicator of your logic, reasoning, and general aptitude. This test not only measures your logical and reasoning skills but shows the hiring manager how you follow directions and timelines and demonstrates your reading comprehension. Companies use an Analytical Reasoning Test to narrow their candidate pool and find the most qualified person to hire. By requiring an Analytical Reasoning Test for employment, companies can ensure that everyone they invite for an interview has the skills needed for a particular role in their company.

What Type of Jobs May Require Candidates to Take an Analytical Reasoning Test?

Jobs that might benefit from an Analytical Reasoning Test include:

  • Law firm staff
  • Office managers
  • Chief executive officers
  • Human resources managers
  • Computer software engineers
  • Financial analysts
  • Sales managers
  • Stockbrokers
  • And many more

What Types of Questions Are on the Analytical Reasoning Test?

Here are a few examples of the types of questions you can encounter on an Analytical Reasoning Test, with explanations for why the correct answer was chosen. Directions: Choose the best answer to the following questions. 1.What is the next number in the sequence? 2, 12, 2, 22, 2, 32, 2, ___ Hint: Carefully follow the pattern, starting from 2.

Correct answer: C

Explanation: The pattern starts with an addition of 10 (2+10=12) then a subtraction of 10 (12-10=2). The following number is an addition of 20 (2+20=22) then a subtraction of 20 (22-2=20). Following this sequence, the next number is 30, followed by 40. So, the correct answer is 42.

2.The Pythagorean Theorem states that a² + b² = c². If a = 7 and b = 11, what does c² equal? Hint: First, find the solution to a² and b². To find the square root of a number, multiply the number by itself.

Correct answer: A .

Explanation: Using the formula, if a = 7, then a² = 49. If b = 11, then b² = 121. 49 + 121 = 170

analytical reasoning problem solving

3.Which of the following images is not a replica of the image on the left in the above diagram? Hint: Look at the color of the boxes and their location.

A. A, B, and C

B. A, C, and D

C. B, C, and D

D. A, B, and D

Correct answer: B

Explanation: First, look at the color of the boxes and their location to determine which is a replica of the image on the left. We find that B is a replica of the image, so B is excluded as an answer to the question. Answer "b" is the only answer that does not have B; therefore, "b" is the correct answer.

Common Questions About Analytical Reasoning Aptitude Tests.

Here are some common questions you might have about the Analytical Reasoning Test:

How quickly do I receive the results of my test? Every company is different. Some may notify you of your results within two to four weeks, and others may not inform you of your results. Some organizations will not give you your score but tell you that you passed if you receive an invitation for an interview.

At what stage in the hiring process is the Analytical Reasoning Test given? After submitting your application and resume, the hiring manager will schedule an Analytical Reasoning Test. Upon successful completion of the test, you will be invited to a telephone or in-person interview.

What is a passing score on the Analytical Reasoning Test? With most companies, a passing score is 80%. However, the higher score you receive will place you higher on the candidacy list, so you want to achieve as high a score as possible.

How do I prepare for the Analytical Reasoning Test? The first step is to take the Mock Questions Analytical Reasoning practice tests. In addition to taking these tests, the best way to prepare is to understand verbal reasoning, non-verbal reasoning, inductive reasoning, and deductive reasoning. Some companies will offer a testing packet with sample questions. If a company provides a testing packet, study the packet before taking the analytical reasoning aptitude test. It is also recommended timing yourself while taking the practice exams to get an idea of how long it will take you to answer each question.

What do I do if I do not know the answer to a question? If you do not know the answer to a question, skip over it and move on to the next question. Since the test is timed, you want to answer as many questions as possible. It is not recommended to leave any questions blank, so after you complete the test, go back to the unanswered questions and choose the answer you believe to be most accurate.

If the questions on the test are based on basic logical knowledge, do I need to study? The questions are based on basic logical knowledge, but it is recommended that you study for the exam if you want to achieve a high score. Those who spend time familiarizing themselves with the content on the test and the imposed time limits tend to feel more confident on the day of the test and have a better chance of achieving the highest possible score.

Helpful Advice for Candidates Preparing for the Analytical Reasoning Aptitude Test.

In addition to studying the materials and taking Mock Questions practice tests, here is some more advice to candidates preparing for the Analytical Reasoning Test:

  • Review logical and analytical reasoning books
  • Take online practice tests
  • Practice using paper and pencil instead of a calculator
  • Watch videos on how to pass an Analytical Reasoning Test
  • Set aside a certain amount of time each day to dedicate to studying test prep books

After completing the Analytical Reasoning practice test, check out the other aptitude tests that Mock Questions offers here .

More Assessment Tests

Cognitive ability test i, cognitive ability test ii, deductive reasoning i, deductive reasoning ii, logic reasoning i, logical perception i.

Complete Test Preparation Inc.

  • Analytical Reasoning Practice Questions with Answer Key

Analytical Reasoning Practice Questions

analytical reasoning problem solving

Analytical reasoning questions, sometimes called AR, assess the ability to look at  qualitative or quantitative information,  such as a group of facts or rules, discern a pattern and conclude what must be true from four multiple-choice options.  Analytical reasoning employ deductive reasoning and generally do not require any specialized or background knowledge.   Examples of the skills analytical reasoning questions use are, comprehending the structure of a set of relationships; recognizing logically equivalent statements; and inferring what could be true or must be true from the given facts or rules.

However, some tests, such as the Canadian Public Service Entrance Test (PSEE) require knowledge of the actual relations between items in order to answer correctly.  Other questions reverse the relationships between nouns or ignore the real relationships to answer correctly.

Answer Key       How to Answer Analytical Reasoning Questions

1. For this question, you must use your knowledge of the real relations between the existing nouns to determine the best response

A CRUX resembles LILO but is closer to the Sun

A TIGO resembles Jupiter but is farther from the Sun

A LILO resembles Earth but is closer to the Sun

Which of the following is the best response?

a. LILO is farther from the Sun than Jupiter

b. CRUX is closer to the Sun than Jupiter

c. Jupiter is closer to the Sun than LILO

d. LILO is farther from the Sun than TIGO

2. For this question, you must use your knowledge of the real relations between the existing nouns to determine the best response

A CUXA is stronger than iron

A BUXA is weaker than wood

A SAMSA is weaker than BUXA

a. CUXA is not the strongest

b. BUXA is weaker than iron

c. SAMSA is not the weakest

d. None of the above

3. For this question, you must use your knowledge of the real relations between the existing nouns to determine the best response

A DJANGO resembles a watermelon but is heavier

A VANGO resembles an apple but is lighter

A TANGO resembles a DJANGO but is heavier

a. DJANGO weighs less than an apple

b. VANGO weighs as much as TANGO

c. TANGO is the heaviest

4. For this question, you must use your knowledge of the real relations between the existing nouns to determine the best response

KAKA is thicker than milk

SUKA is of same thickness as water

BUKA is thinner than water

a. SUKA is the thinnest

b. BUKA is thicker than milk

c. milk is thicker than SUKA

d. KAKA is not the thickest

5. For this question, you must use your knowledge of the real relations between the existing nouns to determine the best response

NONO is longer than meter

BONO is smaller than micron

SONO is longer than BONO

a. BONO is the smallest

b. NONO is the longest

c. SONO is the longest

d. SONO is smaller than nano

6.  For this question, you must reverse the real relation between the existing nouns to determine the best response.

MOLO is longer than lizard

VEL is smaller than crocodile

THOL is longer than VEL

a. VEL is the smallest

b. MOLO is the longest

c. THOL is the longest

d. THOL is smaller than nano

7. For this question, you must reverse the real relation between the existing nouns to determine the best response.

DIMO is colder than Mercury

LOLI is warmer than Neptune

REE is colder than Mercury

a. DIMO is the coldest

b. REE is warmer than DIMO

c. LOLI is the warmest

d. REE is colder than DIMO

8. For this question, you must reverse the real relation between the existing nouns to determine the best response.

SOTA is taller than TATI

KOTI is shorter than giant

SOTA is taller than dwarf

a. TATI is taller than dwarf

b. KOTI is the shortest

c. SOTA is the tallest

d. KOTI and TATI are of same height

9.  For this question, you must reverse the real relation between the existing nouns to determine the best response.

TYGU is faster than bullet

MYGU is of same speed as arrow

JYGU is faster than TYGU

a. bullet is the slowest

b. JYGU is faster than MYGU

c. arrow is the fastest

d. none of the above

10. For this question, you must reverse the real relation between the existing nouns to determine the best response.

RUNI is lighter than PINI

WITI is denser than rubber

PINI is as light as steel

a. PINI is denser than rubber

b. RUNI is not the lightest

c. WITI is the densest

1. B Based on the relations outlined in the first & third statements, we know that a CRUX is closer to the Sun than a LILO, which is closer than Earth. We also know that Earth is closer than Jupiter from the knowledge we have of these existing nouns, and, from the second statement, we know that Jupiter is closer than a TIGO. From closest to farthest, the order of the words is: CRUX, LILO, Earth, Jupiter, TIGO. Therefore, t choice B is the correct answer.

[CRUX<LILO<Earth<Jupiter<TIGO in terms of distance from the Sun] 

Based on real relations, iron is stronger than wood. From the first statement, we know CUXA is stronger than iron which is stronger than wood. From second statement, we know that BUXA is weaker than wood, and therefore weaker than iron and CUXA. From third statement, we know that SAMSA is weaker than BUXA and therefore weaker than other three as well. From weakest to strongest, the order of the words is: SAMSA, BUXA, wood, iron, CUXA. Therefore, choice B is correct.

[SAMSA<BUXA<wood<iron<CUXA in STRENGTH]

3. C A watermelon is heavier than an apple based on real relations. From first statement, we see that DJANGO is heavier than watermelon which is heavier than an apple. From third statement, we know that TANGO is heavier than Django (and therefore heaviest) while, from second statement, we know that VANGO is lighter than apple (and therefore lightest). From lightest to heaviest, the order of the words is VANGO, apple, watermelon, DJANGO, TANGO. Therefore choice C is correct.

[TANGO>DJANGO>watermelon>apple>VANGO in weight]

4. C Based on real relations, milk is thicker than water. From first and second statements, KAKA is thicker than milk, which is thicker than water, which is of same thickness as SUKA. From third statement, BUKA is thinner than water and therefore the lightest. From heaviest to thinnest, the order of the words is KAKA, milk, water & SUKA, BUKA. Therefore choice C is correct.

[KAKA>milk>water=SUKA>BUKA]

5. A Based on real relations, meter is longer than a micron (1 meter = million microns). From first statement, NONO is longer than meter ,which is longer than micron. From second statement, micron is longer than BONO. From third statement, SONO is longer than BONO, making BONO the smallest. Relationship between SONO and NONO is unclear.

Therefore choice A is correct. All other choices are inconclusive.

6. A Reversing real relations, lizard is longer than a crocodile. From first statement, MOLO is longer than lizard, which is longer than crocodile. From second statement, crocodile is longer than VEL. From third statement, THOL is longer than VEL, making VEL the smallest. Relationship between THOL and MOLO is unclear.

Therefore, choice A is correct. All other choices are inconclusive.

7. C Reversing real relations, Mercury is colder than Neptune. From first & third statements, DIMO and REE are both colder than Mercury. From second statement, LOLI is warmer than Neptune which is warmer than Mercury. Mercury is warmer than both DIMO and REE. Therefore LOLI is the warmest. Relationship between DIMO and REE can’t be established.

Therefore, choice C is correct.

8. C Reversing real relations, dwarf is taller than giant. From second statement, KOTI is shorter than giant which is shorter than dwarf. From third statement,  SOTA is taller than dwarf, which is taller than giant and KOTI. From First statement, TATI is shorter than SOTA, which makes SOTA the tallest. The relationship of TATI with others isn’t clear.

9. A Reversing real relations, bullet is slower than arrow. Based on first statement, TYGU is faster than bullet. Based on second statement, MYGU and arrow are of same speed and therefore, faster than bullet. Based on third statement, JYGU is faster than TYGU and therefore faster than bullet. Therefore bullet is slower than everybody and therefore bullet is the slowest. Second and third choices are inconclusive.

Therefore, choice A is correct.

10. C Reversing real relations, steel is lighter than rubber. Based on second statement, WITI is denser than rubber. Based on third statement, PINI and steel are of same density and therefore, lighter than WITI. Based on first statement, RUNI is lighter than PINI and therefore lighter than WITI. Therefore WITI is denser than everybody.

How to Answer Analytical Reasoning Questions

Here are some tips on how to answer analytical reasoning questions:

  • What are the Relationships  Start by identifying the relationships between the nouns. Here, you have three statements that describe relationships between CRUX, TIGO, LILO, Earth, Jupiter, and the Sun. These relationships will be described by words like, “resembles,” “closer to,” “farther from.”
  • Picture the Relationships  Visualize or create a mental diagram of the relationships.
  • CRUX resembles LILO but is closer to the Sun.
  • TIGO resembles Jupiter but is farther from the Sun.
  • LILO resembles Earth but is closer to the Sun.
  • Look for common elements or patterns   Here, you can see that the relationships are objects (CRUX, TIGO, LILO) and planets (Earth, Jupiter) and their positions relative to the Sun (closer or farther).
  • Draw Conclusions What conclusions can you make about the relationships between CRUX, TIGO, LILO, Earth, Jupiter, and the Sun. For example, you can infer that CRUX is similar to Earth, but is closer to the Sun, and TIGO is similar to Jupiter, but is farther from the Sun.
  • Logic Make inferences.  For example, since CRUX is closer to the Sun and LILO is closer to the Sun as well, you can conclude that CRUX and LILO are both closer to the Sun than TIGO (A TIGO resembles Jupiter but is farther from the Sun).    More logic tutorials and practice
  • Double-Check Before finalizing your answer, double-check your answer is consistent with the information provided in the statements.

Example Question (#1)

More practice.

Analytical Reasoning

How to Answer Analogy

Logic and IQ Practice

Graphic Comparisons

Logic Practice II

Sentence Logic

Verbal Classification

Practice for your Test!

Law Enforcement

Correctional Officer BC Police Ontario Police Deputy Sheriff RCMP

Firefighter (Canadian) OTEE PSEE

High School

CCAT (7 & 8)

Watson Glaser

IMAGES

  1. Analytical Thinking and Problem Solving Core Competencies

    analytical reasoning problem solving

  2. Analytical Reasoning Tips (How to solve)

    analytical reasoning problem solving

  3. 17 Analytical Thinking Examples (2024)

    analytical reasoning problem solving

  4. examples of analytical problem solving skills

    analytical reasoning problem solving

  5. Analytical Thinking and Problem Solving Examples

    analytical reasoning problem solving

  6. examples of analytical problem solving skills

    analytical reasoning problem solving

VIDEO

  1. Analytical Reasoning

  2. Logical Reasoning

  3. || IMPORTANT PYQ SOLUTION|| SET02 || SSC| BANK| RAILWAY||BY HIMANSHU SIR||

  4. Analytical Skill Lecture 3

  5. The Art of Critical Thinking: Developing Analytical Skills

  6. ||REASONING PUZZLE| SET 15|UPP

COMMENTS

  1. 59+ Analytical Reasoning Questions and Answers With ...

    Filed Under: Analytical Reasoning. Exam Prep: AIEEE , Bank Exams , CAT , GRE , TOEFL. Job Role: Bank Clerk , Bank PO. 58 19575. Q: What is the number of straight lines and the number of triangles in the given figure. A) 10 straight lines and 34 triangles. B) 9 straight lines and 34 triangles. C) 9 straight lines and 36 triangles.

  2. What Is Analytical Thinking and How Can You Improve It?

    Analytical thinking involves using a systemic approach to make decisions or solve problems. By breaking down information into parts, analytical thinkers can better understand it and come to a sensible conclusion. For instance, once analytical thinkers identify a problem, they typically gather more information, develop possible solutions, test ...

  3. Analytical Reasoning Tests: Free Online Questions & Tips

    Analytical reasoning tests are a go-to tool for employers looking to gauge a candidate's problem-solving prowess. Organizations across a myriad of industries use these assessments to get a handle on the analytical skills that are crucial in the modern workplace.

  4. PDF ANALYTICAL THINKING AND PROBLEM-SOLVING

    If used correctly, analytical thinking will help you: Understand connections between information and occurrences; Recognise the validity of different information and arguments; Recognise the mistakes you make in your thinking and improve; Solve problems and make decisions effectively. 2.1 Importance of Analytical thinking in the workplace.

  5. Study plan for analytical reasoning

    A successful LSAT journey usually consists of a series of many small steps. Before you can get 20 questions right, you'll need to be able to consistently get 10 questions right, then 15, and so on. Don't worry about your scaled score (120-180) right now.

  6. Analytical Thinking, Critical Analysis, and Problem Solving Guide

    Analytical Thinking's Importance in Problem Solving. Certainly! Analytical thinking entails the capacity to gather pertinent information, critically assess evidence, and reach logical conclusions. ... Applying analytical reasoning to work. Assume you are a business owner who wants to boost client happiness. An analytical thinker would collect ...

  7. Analytical Thinking vs Problem Solving: A Comprehensive Comparison

    Problem solving is an essential skill that helps individuals and organizations tackle challenges effectively. Problem-solving skills enable individuals to identify problems or obstacles, analyze the situation, and find appropriate solutions. These skills include critical thinking, analytical reasoning, decision-making, and learning from the ...

  8. Mastering Analytical Thinking: A Guide for Problem Solvers

    Problem-Solving: In business operations, analytical thinking is essential for identifying problems, evaluating potential solutions, and implementing effective strategies to improve efficiency and ...

  9. Free Analytical Reasoning Test Example Questions and Answers

    Almost every important industry employer seeks workers with good analytical reasoning skills, measured by analytical reasoning tests. They help employers find candidates who have the necessary cognitive abilities for learning quickly, adapting, and solving problems. Professionals who take the Analytical Reasoning Tests include. Business analysts.

  10. What Is Analytical Reasoning? A Definition & Overview

    Analytical reasoning is an essential part of management roles, especially those that need critical thinking and problem solving. You might find analytical reasoning most important in careers in law or financial positions, for example. For roles like these, you could encounter analytical reasoning assessments as part of the application process.

  11. Analytical Reasoning Skills Test: Free Questions & Answers

    Analytical reasoning tests measure a candidate's critical thinking and problem-solving skills. Data may be presented in the form of written passages, graphs, tables or shapes. Where questions are based on a series of images, they have much in common with inductive reasoning and non-verbal reasoning tests. Written analytical reasoning ...

  12. Free Analytical Reasoning Test Questions Practice

    Analytical reasoning, also known as logical reasoning, is a problem-solving method that focuses on identifying patterns and using logic to fill in missing pieces. This form of reasoning is slightly more detached from inferences and opinions, and places great emphasis on factual evidence. Analytical reasoning can be further simplified into two ...

  13. What Are Analytical Skills? 9 Examples & Tips to Improve

    8. Problem-solving. Problem-solving appears in all facets of your life — not just work. Effectively finding solutions to any issue takes analysis and logic, and you also need to take initiative with clear action plans. To improve your problem-solving skills, invest in developing visualization, collaboration, and goal-setting skills. 9. Research

  14. Analytical Reasoning Questions: Concepts, Syllabus Topics, Examples

    It can be used in our everyday like in various contexts, such as problem-solving, decision-making, and critical thinking. Analytical reasoning helps individuals make informed choices and understand the underlying principles or logic in a given situation. Examples of the Analytical Reasoning includes the following: Puzzle Solving; Data ...

  15. Problem Solving, Critical Thinking, and Analytical Reasoning Skills

    Critical thinking, analytical reasoning, and problem-solving skills are required to perform well on tasks expected by employers. 1 Having good problem-solving and critical thinking skills can make a major difference in a person's career. 2. ... Analytical reasoning is the ability to look at information and discern patterns within it. "The ...

  16. Analytical Reasoning Questions with Answers: Downloadable PDF

    At Smartkeeda, we recognize the significance of mastering analytical reasoning and offer a comprehensive set of analytical reasoning questions with answers to help you excel in this critical reasoning domain. Mastering analytical reasoning is essential for success in competitive exams, and our analytical reasoning practice questions provide a ...

  17. Analytical Reasoning Tests: Practice Questions & Guide 2024

    An analytical reasoning test is a way of measuring a candidate's comprehension skills and their ability to identify key information, ... and solve problems. Employers often use these tests to evaluate a candidate's problem-solving skills and decision-making abilities. In an academic context, analytical tests are used to assess a student's ...

  18. Analytical Thinking, Problem Solving and Decision Making

    Analytical thinking and problem solving are foundational thinking skills that involve breaking things down into their component parts. They also involve deductive reasoning, drawing conclusions from givens and applying judgments to reach conclusions from a combination of evidence and assumptions. This seminar introduces you to fundamental ...

  19. Analytical thinking & Reasoning: Real-life Examples

    In fact, analytical thinking and analytical reasoning are two very different things, however, related. Analytical thinking is an important aspect of analytical skills. Most of us do not realize how to use analytical thinking and often end up solving the problem incorrectly or half-heartedly.

  20. Analytical Reasoning

    For a cause to be valid, it must be either sufficient or necessary. 6. Syllogism. Syllogism is a part of logical reasoning, especially analytical reasoning. It consists of some statements, and candidates need to derive conclusions from the given statements. The statements and conclusions may seem to be illogical, but while solving questions ...

  21. Analytical Reasoning I

    The Analytical Reasoning Test assesses your logical and critical-thinking skills, analysis, and problem-solving.Most Analytical Reasoning Tests are timed, ranging from 45 to 60 seconds per question, with 20 questions or more. Typically, calculators are not allowed, but you will be able to use a piece of paper and a pencil to calculate ...

  22. Analytical Reasoning Practice Questions with Answer Key

    CRUX is closer to the Sun than Jupiter. c. Jupiter is closer to the Sun than LILO. d. LILO is farther from the Sun than TIGO. 2. For this question, you must use your knowledge of the real relations between the existing nouns to determine the best response. A CUXA is stronger than iron. A BUXA is weaker than wood.

  23. What Are Analytical Skills? Definition and Examples

    For example, you can use communication skills to explain a data visualization to team members and help them understand company performance, or to present high-level findings from a data exercise or statistical analysis. Examples of analytical communication skills include: Verbal communication. Storytelling. Chart, graph, and data presentation.

  24. Boost Logical Reasoning in Manufacturing Engineering

    Finally, reflective practice is a powerful tool for enhancing logical reasoning. After completing a project or solving a problem, take time to reflect on the steps you took and the decisions you made.